Космос и астрономия

Ответить в тред Ответить в тред
Check this out!
Тред тупых вопросов №136 Roche edition Аноним 19/08/20 Срд 00:26:43 5960781
ÉdouardRoche.jpg 37Кб, 298x393
298x393
Rings.jpg 162Кб, 1920x1080
1920x1080
Rochelobes.jpg 218Кб, 844x1089
844x1089
Tidaldisruption.mp4 171Кб, 600x338, 00:00:09
600x338
Тред вопросов о жизни, Вселенной и всём таком.

Спрашиваем то, за что в других местах выдают путёвку в биореактор. Здесь анонимные учёные мирового уровня критически рассмотрят любые гениальные идеи и нарисованные в Paint схемы.

Предыдущий тут: >>591100 (OP)
https://2ch.hk/spc/res/591100.html

Q: Можно быстрее?
A: Можно упасть в пузырь Альбукерке, NASA уже почти надула его.

Q: Я начитался охуительных историй про уфологию, че делать, нам жопа?
A: Да, тебе жопа, можешь сгонять в зогач или куда оттуда пошлют.

Q: Что будет с человеком в вакууме без скафандра / если он упадет на черную дыру / попробует ступить на поверхность газового гиганта/солнца?
A: Он умрёт.

Q: Почему бы не привязать ракету к воздушному шару или стартовать с горы?
A: Космос - это не как высоко, а как быстро, большая часть энергии ракеты уходит на разгон вбок.
Подробнее тут https://what-if.xkcd.com/58/ (английский) https://chtoes.li/orbital-speed/ (перевод)
Аноним 19/08/20 Срд 01:33:09 5960932
Паста про ебаный стыд.
Аноним 19/08/20 Срд 01:50:54 5960983
>>596078 (OP)
>>596093
Постыдный трах...
Первый - Алькубьерре.
Во-вторых, не падайте, а творите вокруг корабля изнутри (иначе не будет родственников).
В-третьих, НАСА искривляет пространство на одну десятую и использует сверхточные интерферометры, чтобы контролировать его вплоть до самого деформирующего исполнительного механизма, как рак в Антарктиде.
Аноним 19/08/20 Срд 02:29:52 5961014
>>596078 (OP)
Разве будет так резко как на видео ломатся.
Аноним 19/08/20 Срд 04:13:36 5961085
>>596073 →
>Не, не смогу, образования недостаточно, только если примерно почувствовать.
>- электроны слишком лёгкие?
>- даже если виртуальную пару растащить и сделать реальной - она сразу анигилируется об соседей?
>- впринципе виртуальную пару ЭМ полем не растащить в две реальные частицы?
>- пока будешь растаскивать и перенаправлять - результирующий импуль погасится в ноль?
>я хз короче.
Почему ЧД не колбасит в разные стороны и почему она не движется никуда от Хокинговского излучения?
Аноним 19/08/20 Срд 06:11:35 5961166
image.png 47Кб, 352x102
352x102
>>596078 (OP)
Система на третьем пике на самом деле циклическая, или там возможен только один такой перенос? Водорода в красном гиганте не так чтобы много, и его остатки ещё сильнее выгорят при эволюции этого нового голубого гиганта.

Алсо, довольно долго тупил в двойной копирайт в углу, лол.
Аноним 19/08/20 Срд 06:36:44 5961187
>>596116
Перефразируй вопрос, пожалуйста. Не очень ясно чего ты хочешь понять.
Аноним 19/08/20 Срд 06:41:45 5961198
>>596068 →
Можно. Проблема в том, что это ничем не лучше ускорения фонариком. Чтобы фармить рабочее тело из вакуума - нужно как-то запасти овердохуя энергии. А если у тебя есть овердохуя энергии - можно ускоряться обычным фотонным лучом неебической мощности.

>>596107 →
>Почему ЧД не движется никуда от Хокинговского излучения?
Потому что фотоны летят во все стороны.
Если привязать к ЧД зеркало с одной стороны (типа как двигатель Шкадова для перемещения звёзд) - она будет ускоряться.
Аноним 19/08/20 Срд 08:06:13 5961249
>>596118
На картинке нарисован перенос массы с красного гиганта слева в звезду справа, которая становится при этом голубым гигантом. Когда звезда справа выгорит и превратится в красного гиганта, пойдёт ли обратный перенос оболочки в звезду слева, которая из-за этого снова станет голубым гигантом? Или из-за изменения состава звёзд такие переносы туда-сюда станут невозможны?
Аноним 19/08/20 Срд 08:21:27 59612710
>>596124
Я вообще не представляю, что ты несёшь, тебе стоит хоть раз релевантную картинку приложить, потому, что у тебя "(C) Slow mass transfer... (c) 2014 Pearson Education Inc." и больше ничего на картинках нет. Хер знает про что ты.

Но если говорить именно о бинарных звездах, то нет, обычно утекание происходит при том моменте когда вторая звезда (по возрасту) расширяется и она дает свою материю на карлик или НЗ/ЧД. Это классика, тогда Новая происходит
Аноним 19/08/20 Срд 08:38:56 59612811
image.png 2186Кб, 844x2390
844x2390
>>596127
Я уже хуй знает, как тебе объяснить. Что тебе не понятно?
Аноним 19/08/20 Срд 08:43:34 59612912
>>596128
Нет, это тебе чего непонятно? Все же понятно нарисовано.
Аноним 19/08/20 Срд 08:46:05 59613013
Вы прослушали очередной выпуск треда тупых вопросов, плавно переходящих в тред тупых ответов. Спасибо за внимание.
Аноним 19/08/20 Срд 09:23:52 59613114
>>596108
Ну так я потому и говорю в полу бублик - в районе жопы создаём поле с градиентом перпендикулярно продольной оси корабля, а потом плавно загибаем его, что бы пары родившиеся в середине развести влево и вправо и по дуге разогнать и выплюнуть назад.

>>596119
А более массивные виртуальные частицы не рождаются, что бы больше смысла ими отталкиваться?
Например Протон/анти Протон.
Можно ещё не только их назад кидать, а ещё немножко к оси доворачивать, что бы был шанс больше что они аннигилируют прямо за кормой и дополнительно толкнут фотоном (или чем там).
Аноним 19/08/20 Срд 09:49:43 59613215
Commonenvelopee[...].png 70Кб, 388x1023
388x1023
Commonenvelopep[...].png 52Кб, 870x1024
870x1024
>>596116
Когда вторая звезда тоже выгорит и заполнит свою полость Роша, ничего никуда перетекать больше не будет, образуется общая оболочка, в которой обе звезды будут тормозиться и падать друг на друга.
Аноним 19/08/20 Срд 10:13:49 59613316
>>596132
Ну не всегда такой сценарий. Есть разные двойные на разных этапах эволюции. Например левая могла догореть до белого карлана, а правый синий станет красным гигантом и начнётся перекат вещества с гелиевой вспышкой.
Аноним 19/08/20 Срд 10:17:45 59613417
>>596132
Ага, ну то есть процесс не зациклен. Я просто думал, что ко времени раздутия правой звезды левая успеет схлопнуться в белый карлик.
Аноним 19/08/20 Срд 10:19:33 59613518
А ещё я буду чаще обновлять тред перед отправкой, ибо >>596133

>>596134
Аноним 19/08/20 Срд 12:15:54 59614319
Лоренцево сокращение увеличивает шансы частицы протунелировать?
Аноним 19/08/20 Срд 13:17:47 59614720
Почему небо синие(голубое)
Аноним 19/08/20 Срд 13:18:19 59614821
Аноним 19/08/20 Срд 14:34:07 59616022
Аноним 19/08/20 Срд 14:35:05 59616123
Аноним # OP 19/08/20 Срд 16:34:42 59617324
Нет желающих в следующий раз перекатить?
Аноним 19/08/20 Срд 16:39:57 59617825
>>596173
Ты здорово справляешься.
Я могу вернуться как это было пару лет назад, не вопрос.
Аноним 19/08/20 Срд 17:04:01 59618326
>Roche
Как это правильно читается – Роке? Роче? Рохе? Роцхе?
Аноним 19/08/20 Срд 17:06:29 59618527
Рыбников прав насчет скорости света?
Аноним 19/08/20 Срд 17:12:49 59618628
Аноним 19/08/20 Срд 17:13:59 59618729
>>596185
Рыбников всегда прав , но не везде.
Аноним 19/08/20 Срд 17:15:43 59618830
Аноним 19/08/20 Срд 17:16:57 59618931
>>596185
Судя по тому, что это какой-то ноунейм - нет. Был бы именитым учёным - можно было бы обсудить. А всякие левые бредовые теории обсуждайте у себя в вацапах или где там сидите.
Аноним 19/08/20 Срд 21:37:12 59620832
>>596078 (OP)
Анимация говно, кстати. Там шарик разваливается, а ведь он задолго до это должен растянуться. И еще раскалиться от трения.
Аноним 19/08/20 Срд 22:21:26 59621733
>>596208
ХУДОЖНИК ТАК ВИДИТ, твои аргументы инвалид.
Аноним 19/08/20 Срд 23:37:38 59624234
>>596078 (OP)
Если я оттолкнусь от МКС в сторону от Земли, меня она всё равно в итоге назад притянет?
Аноним 19/08/20 Срд 23:48:45 59624335
>>596242
Нет, у нее притяжение милипиздрическое. Ты уйдешь на другую орбиту, которая будет пересекаться с орбитой МКС в точке, где оттолкнулся.
Аноним 19/08/20 Срд 23:51:08 59624536
>>596243
И можно будет ухватиться за МКС обратно?
Аноним 19/08/20 Срд 23:52:39 59624637
>>596245
Нет, у вас частота оборотов будет чуток отличаться. Ну, то есть, когда-то может и можно будет, но ты к тому времени мумифицируешься уже.
Аноним 19/08/20 Срд 23:57:55 59624738
>>596242
МКС тебя никуда не притянет, она для этого слишком маленькая. Ты прилетишь обратно либо через виток, либо через полвитка (если оттолкнулся "вбок", например). Если оттолкнешься "вперед" по направлению движения, то хоть обратно и прилетишь, но уже немного позади станции, и будешь отставать с каждым витком. Если "назад", то впереди станции, и никогда до неё не доберешься, возмущения орбиты тебя рано или поздно сведут в атмосферу.

Точно обратно ты никогда не прилетишь, всегда промахнёшься, из-за тех самых возмущений.
Аноним 20/08/20 Чтв 00:08:10 59624839
Вокруг Луны из-за масконов считай нет стабильных орбит. А что с Землей? Или там тут этим можно пренебречь, и атмосфера дает бóльшие погрешности?
Аноним 20/08/20 Чтв 00:14:59 59625040
>>596243
>>596247
Я неправильно сформулировал вопрос, имел ввиду притяжение Земли а не МКС. Спасибо за ответы :3
Аноним 20/08/20 Чтв 01:37:17 59625241
>>596248
Можно пренебречь, Земля гораздо более однородна, плюс её более сильная гравитация и более высокие орбиты спутников практически полностью сводят влияние масконов на нет. У нас тут наоборот, нихуёвые усилия надо приложить, чтобы неоднородности гравитации вообще замерить.
Аноним 20/08/20 Чтв 01:39:14 59625442
>>596252
Эх, никакой романтики!
Аноним 20/08/20 Чтв 04:10:43 59626543
>>596248
Существует общая закономерность - чем тяжелее тело, тем более оно круглое и "жидкое", во всех смыслах. Потому что самогравитация, задающая форму, всё более доминирует над любыми другими силами.

Поэтому тяжёлые планеты в любом случае должны быть более равномерны по внутренней структуре чем легкие. Звезды по той же причине почти совсем круглые по форме (если не вращаются быстро как Ахернар). А нейтронные звезды практически идеально круглые, даже если крутятся сотнями оборотов в секунду.
Аноним 20/08/20 Чтв 04:15:15 59626744
>>596248
>атмосфера дает бóльшие погрешности
Среди возмущений орбиты, примерно до 1500км высоты ионосфера Земли доминирует. Дальше уже Луна и солнечный ветер влияют больше. Возмущения за счет неравномерности геоида по сравнению с эталонным эллипсоидом - совсем маленькие.
Аноним 20/08/20 Чтв 04:21:56 59626845
>>596265
Чем тяжелее и компактней, т.е. ближе к радиусу Ш.
Аноним 20/08/20 Чтв 14:51:00 59630746
>>596252
А как Гоги замерял разницу-то? Как он находясь на орбите может чувствовать притяжение?
Аноним 20/08/20 Чтв 15:07:37 59631047
>>596307
>Как он находясь на орбите может чувствовать притяжение?
Он может чувствовать его изменение. Плотность участка планеты под аппаратом меняется - сверхвысокоточный акселерометр чувствует ускорение в ту или иную сторону.

Ну и естественно, надо уметь компенсировать остальные силы с как минимум не худшей точностью, чем у акселерометра. У гочи эту роль выполнял ёбистый GPS-приёмник, высокоточный стандарт частоты (атомные часы) и бортовой катафот, по которому презиционно измеряли орбиту с земли.

А вообще грубо наблюдать можно по отличию траектории аппарата от предсказанной, например.
Аноним 20/08/20 Чтв 15:08:28 59631148
>>596248
>Вокруг Луны из-за масконов считай нет стабильных орбит
Есть же, LRO собственно по такой и крутится.
Аноним 20/08/20 Чтв 15:13:52 59631349
>>596310
>сверхвысокоточный акселерометр чувствует ускорение в ту или иную сторону.
Каким образом? Он же практически в свободном падении, не отличается по траектории от аппарата в котором расположен.
Аноним 20/08/20 Чтв 15:28:58 59631650
>>596313
Не в 100% свободном, на то она и микрогравитация. При пролёте над повышенной концентрацией массы она на самую писечку создаёт чуть большее притяжение, этого достаточно чтобы акселерометр триггернулся. Да, аппарат быстро придёт в свободное падение опять, но изменение всё равно почуствуется.
Аноним 20/08/20 Чтв 16:16:09 59632351
>>596316
Так он реагирует на повышение гравитации. А будет ли это работать с её уменьшением?
Аноним 20/08/20 Чтв 17:03:53 59632752
>>596323
Ясен красен. Просто в другую сторону.
Аноним 20/08/20 Чтв 18:28:00 59633853
Останется ли от человека вообще что-то, если он как Шепард во втором Масс Эффекте упадет в атмосферу? Прост там дальше было найдено тело Шепарда, хотя он вроде как сгорел упав в атмосферу, когда первая Нормандия была уничтожена внезапно появившимися коллекционерами.
Аноним 20/08/20 Чтв 18:29:50 59633954
>>596338
хуй знает что там за атмосфера была
Аноним 20/08/20 Чтв 18:40:35 59634355
>>596339
https://masseffect.fandom.com/ru/wiki/%D0%90%D0%BB%D0%BA%D0%B5%D1%80%D0%B0

>Кора Алкеры состоит из углерода и водяного льда. Большой размер при малой плотности позволяет планете удерживать толстую атмосферу из метана и аммиака. Считается, что если бы при формировании звёздной системы Алкера обладала бы большей массой, то она стала бы ядром газового гиганта. У Алкеры есть три спутника: Улуру, Ванжинья и Байаме.
Аноним 21/08/20 Птн 16:49:29 59647356
>Согласно исследованиям, Дездемона может столкнуться с Крессидой или Джульеттой через 4—100 млн лет[7].
Речь о спутниках Урана. А как бы выглядело такое столкновение? Как я понимаю, поскольку они движутся в одном направлении, относительная скорость при столкновении будет не очень большой?
Аноним 21/08/20 Птн 18:17:51 59648257
>>596473
забей, это чисто теоретическая хуйня, реальные орбиты спутников Урана сложно предсказать, так как данных слишком мало. Вот если пошлют аппарат, которые пролетит мимо них, померит их массу, резонансы, вращения, массу колец, высчитает всю хуйню - тогда можно будет с точностью говорить.
Аноним 21/08/20 Птн 18:23:34 59648358
>>596482
Меня не что-то конкретное интересует, а просто столкновение двух спутников.
Аноним 21/08/20 Птн 18:30:33 59648459
>>596483
ну хуй знает, даже если они "медленно" будут друг в друга врезаться, тереться друг о друга, то энергия будет такая что они разогреются и развалятся. А если у них орбиты эксцентричные, то там энергии точно будет больше, если один спутник будет разгоняться из апоурания, а другой замедляться из периурания и они столкнутся так. Эти предсказания не рассказывают часто о геометрии столкновений заметь, так как мы геометрию самих спутников почти не знаем, так что там просто крутят точки или шары и потом у них симуляция говорит "1525 раз точки прошли мимо друг друга на расстоянии 1 км, 45 раз точки прошли через друг друга за время такие-то миллионы лет" а может и нет
Аноним 21/08/20 Птн 18:34:07 59648660
1597178111735.png 1729Кб, 1500x1600
1500x1600
Чтобы узнать как они крутят симуляции столкновений, попробуй почитать эту статью
https://perso.imcce.fr/jacques-laskar/pdf/nature08096&s1.pdf

А если еще читать эту статью и одновременно слушать вот этот саундтрек, то воображение разыграется неимоверно.
https://www.youtube.com/watch?v=z_siqjpC6fQ
Хорошие статьи обязательно нужно читать под хорошую соответствующую музыку.
Аноним 21/08/20 Птн 18:51:47 59648961
>>596486
Не, сложна.
>они разогреются и развалятся
То есть не будет чего-то типа слияния ядер в один большой спутник, с разлетающейся шелухой из коры и всяких плохолежащих валунов с поверхности? Все разлетится?
Аноним 21/08/20 Птн 19:03:28 59649062
>>596489
да там дохера факторов - материал из чего они сделаны(один может быть из одного материла, другой из другого), внутренняя структура(есть ли там вообще ядра), геометрия тел(вдруг один спутник это вытянутая игла как оамуамуа, или как арракот состоит из двух частей, а другой спутник это крепыш-ядро), кто в кого врезается, кто тяжелее импактор или цель, вектор и скорость столкновения, имеют ли рядом стоящие спутники приливное воздействие.

Посмотри как мало мы знаем про эти спутники хотя бы на Вики
https://en.wikipedia.org/wiki/Cressida_(moon)
https://en.wikipedia.org/wiki/Juliet_(moon)
конечно в реальности мы можем знать больше чем в википедии написано, но скорее всего это маскимум
Аноним 21/08/20 Птн 23:16:59 59651763
1598041016827.jpg 657Кб, 1090x768
1090x768
Так блять, я не понял, а в чем проблема сделать фотонный двигатель?
Это как ионный, только фотонный.
Берём ядерный реактор на пару сотен мегаватт, используем его энергию для высирания мощного лазера.
Все. Топливо не требуется, реактивное движение без потери массы.

Как сральное ведро, только рабочее
Аноним 21/08/20 Птн 23:55:15 59652364
>>596517
>без потери массы
С потерей массы.

>Берём ядерный реактор на пару сотен мегаватт
И никуда не летим, т.к. на каждый ньютон тяги требуется 300 ебучих мегаватт излучаемого света.
Аноним 21/08/20 Птн 23:55:28 59652465
>>596517
А ты не пробовал посчитать что получится в теории?
Если я не ошибаюсь, то тяга лазера (в ньютонах) равна мощности, деленной на скорость света. Допустим мощность реактора один мегаватт, КПД лазера 100%, а масса всей конструкции одна тонна. Сколько оно будет лететь хоть куда-нибудь?

>реактивное движение без потери массы.
Если только условно. Реально будет масса уменьшаться, иначе никак.
Аноним 22/08/20 Суб 00:19:18 59652866
Где больше рачков, на Европе или Инцеладе?
Аноним 22/08/20 Суб 00:33:10 59652967
>>596338
Зависит от скорости входа, плотности и т.п. На Земле даже угольки не долетят.
Аноним 22/08/20 Суб 00:39:59 59653068
Аноним 22/08/20 Суб 01:36:16 59653169
>>596490
>кто в кого врезается, кто тяжелее импактор или цель
А почему на вот это должно быть не похуй?
Аноним 22/08/20 Суб 02:03:33 59653270
Как там дела у вазимра?
Аноним 22/08/20 Суб 08:25:59 59654071
image.png 3705Кб, 2078x1490
2078x1490
У нас на сраной земляшке корону нельзя увидеть кроме как в случае полного затмения и по новостям в 2020 году из-за атмосферы, как я понимаю, а будет ли ее видно перед рассветом до прихода терминатора (не Шварца) на поверхности планеты/спутника без атмосферы? Такое-то гало поднимается, а потом яркая вспышка и само солнце уже светит?
Аноним 22/08/20 Суб 09:11:50 59654272
>>596524
Если бы я умел считать, то в этом треде бы не сидел
Аноним 22/08/20 Суб 09:26:47 59654473
>>596532
Никак. Он жрёт дохуя, и поэтому ему ёба-аппарат надо пилить. Разве что маленький вариант испытывать.
Аноним 22/08/20 Суб 10:17:14 59654574
>>596531
На дороге видел столкновения легковушки и грузовика? Имеет ли значение масса и кто в кого врезается с какой стороны и скорости?
Аноним 22/08/20 Суб 11:56:16 59655375
>>596517
Нет ни таких реакторов расчитанных на космос, ни целей ни средств.
Если тебе надо дёшево и быстро подскочить внутри СС включая облако Оорта, то пока самая годная идея сначала подлететь как можно ближе к Солнцу а потом развернуть паруса и хуярить на всех парах.
Аноним 22/08/20 Суб 12:14:00 59655576
>>596545
Кто в кого врезается - ваще никакой разницы, важна скорость столкновения, у объектов в космосе вообще нет понятия кто в кого въехал
Аноним 22/08/20 Суб 13:14:36 59655777
>>596555
>Кто в кого врезается - ваще никакой разницы
Для протокола важно.
Аноним 22/08/20 Суб 14:13:34 59655878
>>596555
есть разница на последующие траектории осколков.
Аноним 22/08/20 Суб 14:36:50 59656179
Как измеряют момент инерции космических тел? Масса понятно - по скорости вращения спутника (естественного или искусственного) по данной орбите. А момент инерции?
Аноним 22/08/20 Суб 14:39:19 59656280
>>596517
В том, что реактор не идеален и будет нагреваться. А тепло в вакууме никуда деваться не будет, при этом энергии лазер будет жрать дохуя и долго.
Лучше уж тогда использовать ярд для нагрева водорода, так хоть какая-то тяга будет.
Аноним 22/08/20 Суб 16:34:00 59657181
>>596558
Никакой

>>596557
Ну тщ сержант, там же знака нет
Аноним 22/08/20 Суб 17:42:53 59657682
1598107373269.jpg 59Кб, 1024x875
1024x875
Планирую в будущем купить у Илона Маска билет на Луну, высадиться вблизи американского лунного модуля и взорвать его.

Так вот, будет ли американское правительство преследовать меня по возвращении домой?
Аноним 22/08/20 Суб 17:58:57 59657783
>>596576
>Так вот, будет ли американское правительство преследовать меня по возвращении домой?
Да.
Аноним 22/08/20 Суб 17:59:32 59657884
erosimage.jpg 38Кб, 798x336
798x336
Аноним 22/08/20 Суб 18:00:34 59657985
Аноним 22/08/20 Суб 18:01:11 59658086
Аноним 22/08/20 Суб 18:03:25 59658187
>>596579
Смотри на тень, которую он отбрасывает и на его яркость. Не похоже на камень. Я не ксенфомалити-фаг, который в каждой пупырке видит жизнь, но это очень странная штука.
Аноним 22/08/20 Суб 18:13:26 59658288
Аноним 22/08/20 Суб 18:15:09 59658389
15981083735800.jpg 55Кб, 798x336
798x336
>>596581
Тень ничем не отличается от тени соседних хуиток.

>>596582
Потому что ты взорвал лунный модуль.
Аноним 22/08/20 Суб 18:17:54 59658590
>>596578
>Во-вторых, он стал первым астероидом, у которого появился искусственный спутник, NEAR Shoemaker (в 2000 году), и на который спустя год этот космический аппарат совершил посадку.
Аноним 22/08/20 Суб 18:19:29 59658691
>>596585
а фотку кто сделал тогда по твоему? Астронавт карлан выскочил из аппарата и сфотографировал севший аппарат уже улетая?
Аноним 22/08/20 Суб 18:20:39 59658792
Аноним 22/08/20 Суб 18:21:07 59658893
>>596583
Камень квадратный и имеет рядом присоединённую секцию, которая образует мостик? Это не естественно, особенно учитвая что оно в кратере уже.
Аноним 22/08/20 Суб 18:23:44 59658994
Насколько опасен космос для человека? Вот есть радиация, а что ещё? Мне кажется космос дает четко понять, что нехуй туда лезть, но чего люди все прутся туда? Как вообще наука сейчас решает проблемы безопасности нахождения человека в космосе?
Аноним 22/08/20 Суб 18:27:59 59659195
Аноним 22/08/20 Суб 18:28:20 59659296
>>596589
какой космос ты имеешь в виду? Уточни, где именно в космосе. Нет никакого абстрактного "в космосе", есть много разных конкретных мест в космосе, где условия разные и пока мы не поймем о каком месте ты хочешь услышать - не поймем как отвечать. Есть околоземная, геостационарная, окололунная орбиты, лунная поверхность, солнечная орбита на расстоянии 1 астрономической единицы от Солнца, есть марсианская орбита, орбита Венеры, орбита Меркурия, орбита Юпитера - везде разные условия и радиация.
Аноним 22/08/20 Суб 18:55:17 59659797
>>596578
Это протомолекула.
Аноним 22/08/20 Суб 18:56:33 59659898
>>596589
Россия тоже четко дает понять, что нехуй тут делать, но какие-то долбоебы тут даже живут.
Аноним 22/08/20 Суб 19:29:58 59660199
>>596598
This. Как кто-то вообще додумался жить на территории, где можно кони двинуть, тупо оказавшись на улице в неподходящее время года без средств температурной защиты?
Аноним 22/08/20 Суб 21:08:53 596606100
>>596601
Странный вопрос, однако!
Аноним 22/08/20 Суб 21:15:57 596608101
>>596601
Ты чо сука, против колонизации Марса?
Тебе дали территорию для тренировки.
Аноним 23/08/20 Вск 03:56:08 596648102
Аноним 23/08/20 Вск 10:31:04 596665103
Если подключить к батарее лампочку, то лампочка будет светить и нагреваться.
Если подключить к батарее резистор, то резистор будет нагреваться.
Если подключить к батарее резистор и лампочку, то лампочка будет нагреваться, а резистор нет, хотя лампочка будет светить слабее.

Как резистор умудряется уменьшать напряжение и не нагреваться?
Аноним 23/08/20 Вск 11:28:50 596672104
>>596665
Никак. Он нагревается.
Аноним 23/08/20 Вск 13:12:51 596690105
>>596672
Но нет же, не нагревается
Аноним 23/08/20 Вск 13:19:51 596694106
1598177990132.jpg 109Кб, 1061x1018
1061x1018
Почему космическая промышленность не вкладывает все бабки в разработку более совершенных двигателей с более высоким импульсом?

Те же клиновоздушники находятся на стадии "пререлиз".
Или вот двигатели скайлона.
Или кучи других проектов.

Они там что, не понимают, что если уменьшить стоимость вывода в несколько раз, то это будет уже новая эра в его изучении и освоении?

Аноним 23/08/20 Вск 13:45:21 596698107
Насколько стабильно реликтовое излучение?

Дистанции до источников довольно-таки приличные - подвержено ли оно ощутимому эффекту паралакса в масштабах солнечной системы? На межзвёздном масштабе?

Можно ли выбрать на карте реликтового излучения какие-то наиболее контрастные точки и юзать их для ориентации в пространстве? Практично ли держать на борту такой детектор для ориентации?
Аноним 23/08/20 Вск 15:24:14 596718108
>>596694
Кто тебе, долбоёбу, сказал, что космическая промышленность не вкладывает все бабки в разработку?
Хуею с дебилов.
Аноним 23/08/20 Вск 15:51:23 596723109
>>596690
У тебя погрешность пальца высокая. Попробуй медицинский пирометр.
Аноним 23/08/20 Вск 16:28:57 596728110
>>596694
1. Вкладывают, но химические двигатели почти достигли предела. Идёт разработка детонационных двигателей, например. Сейбр не очевидно как запилить. Других проектов нет особо, это все слабореализуемые по разным причинам фантазии.
2. Клиновоздушники это мем, они на самом деле хуевые - тяжелые, горячие и имеют смысл только в SSTO с вертикальным взлётом, а SSTO всегда хуже многоступенчатой схемы. В многоступенчатой же можно повесить два разных движка, и - о ужас - две топливных пары (тяговую внизу, высокоимпульсную вверху).
3. "Уменьшение стоимости вывода" - точно такой же мем. Аппараты стоят дороже пусков даже сейчас, и всегда будут стоить достаточно дорого, просто потому что в космосе летать сложно.
Аноним 23/08/20 Вск 19:11:03 596749111
>>596698
Интересный вопрос
Аноним 23/08/20 Вск 19:25:32 596753112
cobe53ghz.gif 100Кб, 612x792
612x792
>>596698
Очень стабильно (колебания в считанные микрокельвины), это ж изначально излучение газа, равномерно заполняющего всю Вселенную. Никакого параллакса нет, контрастных точек нет, ориентироваться по нему невозможно, да и даже пытаться это делать — быть дауном, тебе ж дали отовсюду видные пульсары, ориентируйся по ним.
Аноним 23/08/20 Вск 19:29:30 596754113
1598200168793.jpg 389Кб, 1080x1255
1080x1255
>>596698
Технически это возможно
Аноним 23/08/20 Вск 19:58:22 596759114
Какая программа покажет нашу Галактику с планетами, а желательно и всю Вселенную в движении?
Аноним 23/08/20 Вск 20:54:29 596763115
Можно ли теоретически сделать космо-телескоп с существующими перспективными технологиями, который не сломается и не закончится даже через 1000 лет?
Аноним 23/08/20 Вск 21:00:51 596764116
>>596763
Нет, нельзя.

Космос - место опасное, огромную чугуниевую йобу туда не выведешь, от малоточных приборов толка нет, а любая точная аппаратура сложна и ненадежна даже в условиях земляшки.

Но можно летать на шатоле и чинить.
Аноним 23/08/20 Вск 23:51:24 596775117
>>596764
>можно летать на шатоле и чинить.
Не нужно. Вместо этого нужно сразу же вместе с телескопом пускать 2 дронов роботов с руками, которые к нему пристыкованы и могут отстыковаться, осмотреть, сами поченить.
Аноним 23/08/20 Вск 23:58:40 596776118
>>596775
Лучше тогда орбитальную СТО+АЗС с дронами ремонтниками и заправщиками, телескоп то там не один болтается.
Аноним 24/08/20 Пнд 00:23:03 596777119
>>596776
Смотря как далеко. Какой-нибудь телескоп в точке фокусировки солнца скорее всего будет один.
Аноним 24/08/20 Пнд 00:35:43 596779120
Аноним 24/08/20 Пнд 01:16:01 596781121
>>596779
Первую, может там поняшки есть.
Аноним 24/08/20 Пнд 16:30:48 596863122
Посоветуйте какие-нибудь годные лекции/курсы/видосы на тему астрономии, вот этого космоса вашего, чтобы начать понимать, что там вообще происходит и как всё работает.
Аноним 24/08/20 Пнд 17:26:41 596872123
>>596863
Хочешь понравиться одной девочке?
Аноним 24/08/20 Пнд 18:00:23 596878124
>>596872
А может мальчику. Давай лучше видосы кидай.
Аноним 24/08/20 Пнд 18:08:35 596879125
>>596775
А если ремонт сложный, типа замены линзы?
Аноним 24/08/20 Пнд 20:16:34 596890126
>>596879
Проще новый телескоп в таком случае запустить.
Аноним 25/08/20 Втр 06:47:58 596927127
index.jpg 7Кб, 225x225
225x225
Увеличится ли подъемная сила воздушного шара, накачиваемого горячим воздухом, при снижении температуры окружающей среды?
Можно ли будет, например на титане летать на шаре наполненном воздухом комнатной температуры?
Аноним 25/08/20 Втр 08:48:21 596937128
>>596927
Да. Чем холоднее вокруг, тем выше плотность воздуха.
Аноним 25/08/20 Втр 16:35:44 596959129
image.png 2019Кб, 1149x893
1149x893
Почему у него сопла яркие внутри?
Аноним 25/08/20 Втр 16:39:59 596961130
)
Аноним 25/08/20 Втр 17:11:17 596965131
>>596959
Потому что так нарисовали.
Аноним 25/08/20 Втр 17:23:06 596966132
>>596965
Это же фотография
Аноним 25/08/20 Втр 19:07:43 596971133
Всем ку, добра, позитива etc
Вот я слышал, что наша вселенная может быть одной из множества других со своими законами физики и жизнью(её отсутствием)
Вопрос - Эти вселенные(очевидно включая и нашу) находятся в каком-то пространстве, как планеты, звёзды, черные дыры внутри нашей вселенной?
Ну, логически это может быть так?
Аноним 25/08/20 Втр 19:32:35 596974134
image.png 145Кб, 721x337
721x337
image.png 69Кб, 300x320
300x320
>>596753
>этот иньянь
Китаезы о чем-то догадывались?
Аноним 25/08/20 Втр 19:38:15 596975135
Аноним 25/08/20 Втр 19:50:58 596980136
>>596974
Нет. Это развёртка сферы на плоскость. В 3Д это выглядит как шар у которого половина синяя половина красная. Можешь сделать другую развёртку будет другая картина.
Аноним 25/08/20 Втр 19:59:19 596982137
4r.gif 178Кб, 625x200
625x200
>>596971
>находятся в каком-то пространстве
Да.
>как планеты, звёзды, черные дыры внутри нашей вселенной
Нет.
Аноним 25/08/20 Втр 20:01:12 596984138
Аноним 25/08/20 Втр 21:09:23 596993139
Вопрос про ВРЕМЯ
Сутки на Земле определяются 24 часами. Но почему-то именно 24, а не 100, 13 или 42.
Помнится, кто-то мне объяснял, что тут всё хитро завязано на математике и геометрии. То есть, даже если наша планета вдруг остановится, или наоборот начнёт наворачивать круги вокруг своей оси разительно быстрее, то всё равно будет 24 часовых пояса и время на 24 часа, просто планет больше не будет в них укладываться.
И вот читаю я рассказы про далёкие миры, играюв игры, смотрю фильмы-мультфильмы, и меня вё будоражит этот вопрос.
Почему 24-то?
На других планетах, получается, можно вывести формулу времени тоже основываясь на математике? Как? Если я говорю, что есть планета с сутками на 31 час или планета с сутками на 13, то это не потому что вне зависимости от размера, они вращаются одна быстрее, другая медленнее, а потому что у них математически другие параметры? Это как-то связано с тем, что метр на земле выводится тоже математически?
Аноним 25/08/20 Втр 21:14:15 596994140
image.png 301Кб, 955x955
955x955
>>596993
>24 часовых пояса
Их 37.
>Почему 24-то?
Так ПРИНЯТО. Все, больше нет причин.
Французы лет двести назад хотели метрическое десятичное время ввести 10:100:100 но не срослось, к сожалению.
Секунда была бы чуть быстрее.

На других планетах будут свои часы.
Аноним 25/08/20 Втр 21:14:44 596995141
>>596994
Слово "метрическое" забыл зачеркнуть.

быстрофикс
Аноним 25/08/20 Втр 21:40:26 596999142
>>596993
Ето не математика, ето так получилось. Шумерская, что ли, система счисления, оттуда все эти дюжины и прут (12 x 2 = 24 часа, 12 x 5 = 60 минут). Все просто спопугайничали, а потом менять было уже поперек шерсти. У леворюционных французов, вон, не получилось.
Аноним 25/08/20 Втр 21:47:20 597001143
>>596763
Нельзя, в спутнике есть и электроника, постепенно набирающая дозу и её может прошибить тяжелой частицей, и механика (например маховики ориентации), и память с ограниченным сроком жизни, и много чего ещё нестабильного, те же материалы - космос весьма агрессивная среда в целом. В общем лет 50 аппарат прожить вполне способен. 1000 - вряд ли, даже если его под это специально заточить.

Ну и 1000 лет ему просто не нужны, т.к. уже через несколько десятков всё нахуй может поменяться.

>>596879
1. У больших телескопов нет линз, у них зеркала. Зеркала легче и их проще делать при больших диаметрах.
2. Главное зеркало заменить не сильно сложнее, чем бортовой компьютер, преобразователь напряжения или маховик.
3. Но оно составляет большую часть цены аппарата, поэтому смысла мало.
Аноним 25/08/20 Втр 22:08:59 597003144
>>596999
Всегда было интересно, как так вышло, что эта система одна на всю землю. Правда не настолько интересно, чтобы это нормально погуглить. Были ли другие какие-то системы измерения времени, которые потом были вытеснены?
Аноним 25/08/20 Втр 22:15:17 597005145
Аноним 25/08/20 Втр 22:15:27 597006146
>>597003
Римасосы распространили вместе с футами и лигами.
Аноним 25/08/20 Втр 23:32:41 597011147
Щас видос какой-то посмотрел, нихуя не понял. Короче, вот в центре галактики находится черная дыра, да? А ещё там было что-то про квазар. Типа они пронизывают вселенную своим всплеском энергии как будто как из середины тоже. То есть, квазар и черная дыра находятся вместе посередине? Если да, то почему черная дыра её не засасывает?
Аноним 25/08/20 Втр 23:34:41 597013148
Что такое инфляция? Схуяли она появилась и закончилась? Не является темная энергия инфляцией? Не является ли инфляция костылем ебаным, чтоб теория не развалилась?
Аноним 25/08/20 Втр 23:36:06 597015149

>>597006
>Римасосы распространили вместе с футами и лигами
>распространили вместе с футами
>с футами
mein respekturen
люблю футанари
Аноним 25/08/20 Втр 23:37:05 597016150
mrchangsenorcha[...].gif 954Кб, 285x235
285x235
Аноним 25/08/20 Втр 23:39:39 597017151
image.png 2256Кб, 1920x1080
1920x1080
>>597011
Квазар и есть черная дыра.
Когда у нее много ЖРАТ, она фонит шо твой экран мобилочки посредь ночи.
Такое было на заре формирования голактек, когда они были молодые, шутливые и жестокие.
Давно все устаканилось и современные галактики без квазаров, черные дыры посередке только чутка хавают звезды, и потому фонят слабо.
Квазары видны те, которые были давным далеко, еще с тех времен.
Аноним 25/08/20 Втр 23:47:20 597019152
Можете пояснить, почему луна улетает от земли, а титан от сатурна? Я про орбитальную механику знаю только с ksp и там если на обьект не действует никакая сторонняя сила, то он будет на своей орбите вечно.
То что тогда двигает луну от своей планеты?
Аноним 25/08/20 Втр 23:48:29 597020153
>>597019
Земные приливы ускоряют ее.
Аноним 25/08/20 Втр 23:55:15 597021154
>>597020
мда, ебать я дибил, в следущей же минуте видео поясняется причины отдаления что луны, что титана. правда, я так и не понял как именно приливные силы влияют, ну да и ладно
Аноним 25/08/20 Втр 23:58:02 597023155
image.png 30Кб, 312x169
312x169
>>597021
>правда, я так и не понял как именно приливные силы влияют, ну да и ладно
Земля вращается быстрее, таща с собой приливы, в частности с помощью суши, луну утаскивает за ними, передавая ей часть момента.
Это убивает краба замедляет вращение земли.
Аноним 26/08/20 Срд 00:08:52 597024156
>>597023
на картинке нихуя не понятно, но смог представить, как это работает. Спасибо за пояснение.
Аноним 26/08/20 Срд 00:09:43 597025157
>>597024
Соболезную.
Это самая простая и понятная схема которая идеально объясняет феномен.
Аноним 26/08/20 Срд 00:12:05 597026158
Правда, что при наличии дыхательной маски с баллоном кислорода и теплой одежды на Титане можно выжить без герметичного скафандра?
Аноним 26/08/20 Срд 00:13:33 597027159
>>597026
>−179.5 °C
Ты где такую одежду видел-то?
Аноним 26/08/20 Срд 00:14:23 597028160
>>597024
Луна поднимает прилив. Но у земной поверхности есть нехуевая вязкость, поэтому прилив "сразу" не сгладится, если убрать создавшую его силу. Земля вращается быстрее, чем Луна по орбите, поэтому приливный горб убегает из-под Луны вперед (он "не успевает" разгладиться, см. предыдущее предложение). Приливный горб притягивает к себе Луну, а поскольку он "впереди", то он придает ей ускорение, заставляя карабкаться выше.
Аноним 26/08/20 Срд 00:15:02 597029161
>>597025
Тв я и без нее понял. Просто на картинке нет суши, которая толкает воду, да и сама вода изображена фигово.
Аноним 26/08/20 Срд 00:15:24 597030162
>>597028
>он придает ей ускорение, заставляя карабкаться выше.
Он огурцачер, он понимает, что ускорение поднимает орбиту.
Аноним 26/08/20 Срд 00:26:27 597031163
image.png 46Кб, 922x819
922x819
Вот более понятная картинка с водой и землей
Аноним 26/08/20 Срд 00:33:42 597033164
image.png 729Кб, 924x3141
924x3141
>>597031
НАЧАЛЬНИК ПО ОРБИТАЛЬНОЙ МЕХАНИКЕ УШЕЛ НА ОБЕД
@
ЛУНА НАМАТЫВАЕТСЯ НА ЗЕМЛЮ
@
ТИТАН ЛЕЗЕТ В УРАНУС
@
ПЛУТОН ОБЪЯВЛЯЮТ ПЛАНЕТОЙ
@
НА ОРБИТУ ЗАПУСКАЮТ ПЛАНЕТУ ИЗ МОЛЯ КРОТОВ
Аноним 26/08/20 Срд 00:52:35 597034165
>>597027
Хорошо, очень теплой одежды.
Аноним 26/08/20 Срд 00:55:51 597035166
>>597034
Ты только что скафандр.
Аноним 26/08/20 Срд 01:00:18 597036167
>>597026
>>597027
Ну если только температура беспокоит - то почему бы и нет, из какой-нибудь керамоткани одежда вполне выдержит и изолировать сможет. И на еблет нужны очёчи/маска. По сути это будет слабо отличаться от скафандра.

Главная проблема в том, что в атмосфере Титана есть всякая шняга из цианогруппы, отравишься как нехуй. И вообще много плохо пахнущих и воздействующих на человека углеводородов. Так что только герметичный скафандр и отмываться в шлюзе.
Аноним 26/08/20 Срд 01:01:14 597037168
>>597036
>очёчи
Ты что, ДУРАЧЕК, в этом слове "ё" использовать?
Аноним 26/08/20 Срд 01:02:42 597038169
>>597037
Опять ты выходишь на связь, поддельный граммарнаци? Ё там.
Аноним 26/08/20 Срд 01:03:51 597039170
>>597038
ОчОчки.

После ж, ч, ш, щ для передачи ударного гласного о пишется буква о или ё. Буква о пишется в следующих случаях.

На месте беглого гласного о в именах существительных и прилагательных, напр.: жор, обжора, прожорливый (ср. жрать), жом (ср. жму), ожог, поджог, пережог, изжога (ср. жгу, жгла); рожон (ср. род. п. рожна), шов (шва); княжон (род. п. мн. ч. от княжна), ножон (род. п. от ножны — устар. вариант слова ножны), мошон, мошонка (род. п. мн. ч. и уменьш. от мошна), кишок, кишочки (род. п. и уменьш. от кишки), квашонка (уменьш. от квашня), кошомка и кошомный (от кошма), очочки (от очки), очочко (от очко),
Аноним 26/08/20 Срд 01:06:33 597040171
>>597039
Да нет же, это разные слова! Очёчи - придуманное, но устойчивое слово, а не уменьшительное от "очки". И пишется именно так, поди найди мне "очочи" к примеру. а потом поищи "очёчи" и охуей
Аноним 26/08/20 Срд 01:08:48 597041172
>>597040
Зачем мне искать подтверждение неграмотности быдла, я это и так знаю.
Правило перечитывай до просветления, тут "о", это объективный факт, выключай манёвры и извинись. https://orfhographia.ru/orfografia.php?sid=17#pp17
Аноним 26/08/20 Срд 01:12:59 597042173
>>597041
>Зачем мне искать подтверждение неграмотности быдла, я это и так знаю.
Так "очочи" ты нигде не найдёшь, потому что нет такого слова. Вообще. Очёчи - обиходная игра со словообразованием, устойчивый эрратив если хочешь.
Ну и лингвистика - наука описательная. Говорят так, как говорят, а лингвисты едва успевают переставлять кофе в средний род.
Аноним 26/08/20 Срд 01:16:51 597043174
>>597042
>очочки
>корень -очоч-
>суффикс -к-
>окончание -и-
>очочи
>корень -очоч-
>окончание -и-
Все по тем же правилам.
Или у тебя новое слово с корнем -очёч-, либо ты насмотрелся на неграмотное быдло и зачем-то на него равняешься.
Аноним 26/08/20 Срд 01:21:27 597044175
Screenshot (255).png 3Кб, 302x151
302x151
Screenshot (257).png 4Кб, 312x205
312x205
>>597043
Я думаю у тебя проблемы с пониманием написанного.
https://ru.wikipedia.org/wiki/Эрратив
Свидетельство употребляемости - на пикчах. Или ты удаффком тоже по правилам будешь разбирать?
Аноним 26/08/20 Срд 01:22:36 597045176
>>597044
>Дебилы используют неграмотное свежевыдуманное слово.
С этим фактом я не спорил. От этого грамотным оно не становится. Пиши его правильно, вот и всё.
Аноним 26/08/20 Срд 01:26:26 597046177
dv.png 671Кб, 4960x3508
4960x3508
>>597033
Не шаришь в орбитальной механике. Во первых, орбита сменится как на первом пике, если дать определенную тягу точно к земле.
Во вторых, так как тяга ОЧЕНЬ маленькая, то и будущая траектория постоянно см пик 2 меняется, а поэтому этим вектором можно пренебречь. А вот вектором в сторону движения, который как раз есть, уже не пренебрежешь, поэтому луна и будет ускоряться, а следовательно и увеличивать свою орбиту.
Аноним 26/08/20 Срд 01:27:34 597047178
>>597046
почему то только второй пик загрузился, но так как он продолжения первого, то пох.
Аноним 26/08/20 Срд 01:28:34 597048179
>>597046
>отвечать на явно юмористический пост
>на картинке нарисована веревка которая наматывает луну
А ты НЕ САМЫЙ НАДЕЖНЫЙ ТВЕРДОТОПЛИВНЫЙ УСКОРИТЕЛЬ НА СТАРТОВОЙ ПЛОЩАДКЕ, да?
Аноним 26/08/20 Срд 01:31:35 597049180
>>597038
>>597040
>>597042
>>597044
Я, пожалуй, извинюсь, за доёбку. Слово глаза режет из-за неграмотности, но оно действительно в ходу.
Аноним 26/08/20 Срд 01:33:40 597050181
>>597048
Соре, не шарю в космическом самосборе.
Аноним 26/08/20 Срд 02:05:04 597051182
image.png 3296Кб, 3836x2154
3836x2154
Какие бывают наблюдаемые квантовые эффекты в макромире?
Аноним 26/08/20 Срд 02:08:13 597052183
>>597051
Существование полупроводниковой электроники.
Аноним 26/08/20 Срд 02:11:11 597053184
>>597052
Лучше бы КВД упомянул, он хотя бы воочию наблюдается.
Фиг знает, полупроводники - это эффекты в микромире.
Аноним 26/08/20 Срд 03:51:38 597054185
>>597045
>От этого грамотным оно не становится.
Но это не точно.
Аноним 26/08/20 Срд 05:54:26 597055186
15982030890560.png 453Кб, 600x337
600x337
>>597003
>>596999

Моя манягипотеза, что все дело в числе 4. Если у тебя вместо цифр орехи/камешки/палочки/черточки, это будет наибольшее количество, которое глаз схватывает без деления на части, целиком, на лету. Пять палок мы уже разбиваем на 2 и 3, это не заметно, потому что очень быстро (к тому же десятичная перепрошивка). Циферблата кстати напрямую касается.
Наверно в какой-нибудь гештальпсихологии должны быть пруфы, а мой любимый филологический:

2 овцы - 3 овцы - 4 овцы - 5 овец
2 мешка - 3 мешка - 4 мешка - 5 мешков
2 солдата - 3 солдата - 4 солдата - 5 солдат
2 спейсача - 3 спейсача - 4 спейсача - 5 спейсачей

То есть в русском в итоге вывелось различие между "несколько" и "дохуя". А у шумеров в итоге вывелось число, делящееся на 4 и достаточное для хозяйственных нужд. инб4 пять пальцев - дохуя у тебя в жизни ситуаций, когда надо сосчитать пальцы? вот и у шумеров не дохуя.
Аноним 26/08/20 Срд 07:40:56 597060187
Почему черные дыры не зарастают?
Аноним 26/08/20 Срд 07:43:30 597061188
>>597060
Они "зарастают" в теории благодаря излучению Хокинга, но испарение занимает бесчисленное количество времени.
Это область пространства-времени обусловленная колоссальной гравитацией, эта масса находится на дне гравитационного колодца и ей не выбраться с него, т.к. даже свет не может покинуть эту область.
Аноним 26/08/20 Срд 09:57:18 597072189
Почему международный стандарт массы КИЛОграмм? Почему все единицы СИ это %единицанейм% а масса это кило+%единицанейм%? Почему не взяли Мась условную за стандарт без метрической приставки и уже от нее плясали? Был бы у нас Мась как килограмм, миллиМась как грамм и килоМась как тонна.
Как-то логика не укладывается.
Понятно, почему килограмм взяли, литр воды измерить легко, но почему он кило- то?
Аноним 26/08/20 Срд 12:34:09 597088190
>>597072
Потому что грамм звучало как граф во французком. А это по буржуйски и негоже в равноправной, пост-революционной франции иметь меру более высокую по статусу, чем другие. Вот и добавили приставку кило. А потом прижилось.
Аноним 26/08/20 Срд 12:51:52 597092191
Аноним 26/08/20 Срд 13:19:19 597093192
после 19 перезв[...].png 30Кб, 970x1124
970x1124
Может ли быть крупная двойная луна у планеты? Например, у планеты уровня Уран и больше (будь то Сатурн или Юпитер), спутник уровня Луны (земной) или вообще Титана, но спутник двойной, вращающийся парой, центр масс пары которого крутится по орбите родительской планеты?
Аноним 26/08/20 Срд 13:21:06 597094193
>>597093
Очень далеко от планеты. Не в нашей солнечной системе.
Аноним 26/08/20 Срд 14:12:08 597098194
>>597093
Такое разрушится приливными силами. Тот же эффект, благодаря которому почти все крупные спутники находятся в приливном захвате (когда оборот вокруг своей оси равен по продолжительности обороту вокруг центрального тела) и обращены к планете одной стороной. Чем дальше от центрального тела, и чем меньше расстояние в паре, тем эффект слабее, но все же сомневаюсь, что такая парочка переживет 4 миллиарда лет в масштабах планета - спутники (вот Солнце - планеты может).
Аноним 26/08/20 Срд 16:39:07 597109195
>>597093
Схема взаимодействия Януса и Эпиметея.жпг
Аноним 26/08/20 Срд 17:00:01 597113196
gaaaaaayyyy.png 89Кб, 500x500
500x500
Аноним 26/08/20 Срд 18:10:32 597118197
>>597109
Во-первых, эти пельмешки не сравнить с размером Луны или Титана с Ганимедом. Во-вторых, ЭТО космическое ДРУГОЕ - у них ведь не общий центр масс, а нахлёстная рокировочная орбита
>>597098
>Тот же эффект, благодаря которому почти все крупные спутники находятся в приливном захвате (когда оборот вокруг своей оси равен по продолжительности обороту вокруг центрального тела) и обращены к планете одной стороной.
Мне почему-то кажется, что вращательный момент в такой паре будет более сильным и влиятельным, чем стопорящий эффект, вызывающий приливный захват
Аноним 26/08/20 Срд 19:55:17 597128198
Напомните пож какой угол плоскости вращения Земли вокруг Солнца к плоскости диска Млечного Пути, если есть картинку буду благодарен.
Аноним 26/08/20 Срд 19:58:54 597130199
image.png 2034Кб, 1200x900
1200x900
image.png 1124Кб, 1399x712
1399x712
image.png 272Кб, 720x260
720x260
image.png 125Кб, 600x600
600x600
image.png 78Кб, 579x319
579x319
image.png 40Кб, 946x191
946x191
Аноним 26/08/20 Срд 20:15:59 597134200
>>597130
Здорово, то есть в летнее солнцестояние Земля между Солнцем и центром Галактики на прямой линии, если правильно понял.
Аноним 26/08/20 Срд 20:20:08 597136201
>>597134
На самом деле наоборот. См. 4 пикчу. После осеннего равноденствия мы оказываемся в Стрельце, примерно во время зимнего солнцестояния.
Первая пикча не такая информативная, четвертая лучшая в наборе, на нее ориентируйся.
Аноним 26/08/20 Срд 20:21:30 597137202
>>597136
DISREGARD THAT I SUCK COCKS

>>597134
Да-да, ты все правильно понял.
Аноним 27/08/20 Чтв 03:17:59 597176203
Сколько всего галактик в видимой и невидимой части ?
Аноним 27/08/20 Чтв 12:13:56 597215204
Аноним 27/08/20 Чтв 14:01:55 597236205
Помнится, кто-то писал что на поверхности Сатурна g таково, что его не отличить от земного. Это навело меня на мысль о создании сеттинга на поверхности небольшого газового гиганта (не обязательно самого Сатурна, можно планету и поменьше), где сама планета по факту газовый гигант, но у неё на поверхности почему-то есть поверхность. Твёрдая, каменистая, с растительностью и водой, то есть обитаемая, но там корочка коры планеты небольшая, пара километров, и каким-то научно обоснованным образом, чем ниже ты бы продвигался по толщине этой коры, тем более рыхлой становилась бы толща, а потом, в самом низу коры, где кора резко обрывается и начинается условная тёплая пустота, ведущая вниз, к ядру, и конвекционными потоками оттуда поднимается лёгкая пыль и наливает на кору. Местами иногда куски налипшего падают обратно вниз, в ядре "перевариваются", а затем пылью обратно липнут к поверхности со внутренней стороны. Ну и вопрос - какого размера тогда должна быть планета, чтобы получить при критерии гравитация на поверхности как на Земле плюс-минус?
Луны у неё могут быть? Повлияли бы?
Аноним 27/08/20 Чтв 18:50:18 597268206
>>597019
> Можете пояснить, почему луна улетает от земли, а титан от сатурна?
Земля - это не конкретная точка с гравитацией. Это охуенных размеров сборище точек, сиречь элементарных частиц, у каждой свое притяжение, и каждая отдельно взятая часть планеты тоже обладает своей гравитацией. Просто если в небольших по времени масштабах это можно считать единой системой, то в более крупных Земля вертится и передает энергию вращения с более близкой к Луне стороны на саму Луну, перемещая ее на более высокоэнергетическую орбиту - то есть, повыше. Со временем Луна замедлит Землю настолько, что будет висеть на новом земном геостате и ускоряться перестанет. Но к тому моменту она с куда большей вероятностью улетит в ебеня либо погибнет при расширении Солнца.
Аноним 27/08/20 Чтв 22:38:28 597282207
изображение.png 13Кб, 791x281
791x281
В небе вижу пикрил. Что обведено красным и синим? Вроде как синее это юпитер, не помню точно, но вот красное?
Аноним 27/08/20 Чтв 23:00:22 597284208
image.png 970Кб, 1560x780
1560x780
>>597236
>поверхности Сатурна
Это где она такая?
> Это навело меня на мысль о создании сеттинга на поверхности небольшого газового гиганта
Ты только что Беспин.
Аноним 27/08/20 Чтв 23:08:37 597285209
>>597236
>Ну и вопрос - какого размера тогда должна быть планета, чтобы получить при критерии гравитация на поверхности как на Земле плюс-минус?
Гравитация планеты не зависит от ее размеров. Существуют такие планеты, которые размером с землю, но при этом имеют чудовищную гравитацию, что там прямоходящие никогда не появятся.
Аноним 27/08/20 Чтв 23:37:27 597289210
>>597285
>Гравитация планеты не зависит от ее размеров
Эм, чтоа? Обратно пропорциональна квадрату радиуса, прямо пропорциональна массе же (для достаточно сферически-симметричных планет, на практике радиуса в ~250 км уже достаточно для этого).
Аноним 27/08/20 Чтв 23:39:20 597290211
Посчитайте, пожалста, какого радиуса будет планета из чистого осмия чтоб на поверхности был 1Г?
Аноним 27/08/20 Чтв 23:43:11 597291212
Аноним 27/08/20 Чтв 23:44:21 597292213
>>597291
Из кротов будет планета-бульон.
Аноним 27/08/20 Чтв 23:52:50 597294214
>>597290
Объем шара равен 4/3 π r^3. Масса: М = 4/3 π r^3 ρ. Гравитация на поверхности будет G M / r^2 = 4/3 π G r ρ. Подставляешь значения констант, решаешь относительно r.
Аноним 27/08/20 Чтв 23:56:26 597295215
Аноним 27/08/20 Чтв 23:57:46 597296216
>>597295
У меня получилось r ~ 1550 км. Кто из нас ошибся на порядок?
Аноним 28/08/20 Птн 00:00:47 597297217
>>597296
Я. Ноль проебал.
1552 км
Аноним 28/08/20 Птн 00:03:17 597298218
Ради интереса, для кротов у меня получился радиус около 35 000 км (предположения: плотность крота примерно равна плотности воды, кроты несжимаемы).
Аноним 28/08/20 Птн 00:13:26 597299219
>>596561
Бамп вопросу. Вот откуда он возник:
>Предположительно Ганимед состоит из трёх слоёв: расплавленного ядра из железа или сульфида железа, силикатной мантии и внешнего слоя льда[5][51] толщиной 900—950 километров. В пользу этой модели свидетельствует малый момент инерции, измеренный во время облёта Ганимеда «Галилео» — (0,3105 ± 0,0028)×mr2[5][51] (момент инерции однородного шара равен 0,4×mr2, а меньшее значение коэффициента в этой формуле говорит о том, что плотность растёт с глубиной).
Аноним 28/08/20 Птн 00:40:57 597302220
>>597296
>>597297
Немного меньше луны, а гравитация как на Земле, охуеть.
А будет у этого осмиевого шара тектоника? Точнее так - она может быть?
Аноним 28/08/20 Птн 01:42:23 597317221
Аноним 28/08/20 Птн 02:08:15 597320222
>>597295
>>597296
>>597297
А сколько брать плотности у Сатурна, чтобы 1g получилось у него на поверхности?
Аноним 28/08/20 Птн 02:29:26 597322223
>>597302
Тектоника возникает не от состава, а от какого-то источника энергии, когда изнутри прет тепло, растапливающее все нахуй, а снаружи планета застывает, потому что получает от Солнца недостаточно энергии. Так что надо либо брать слаборадиоактивный изотоп осмия (кандидат тут только осмий-186), либо вешать шарик спутником на орбиту с заметным эксцентриситетом и/или наклонением к экватору центрального тела.
Аноним 28/08/20 Птн 07:53:28 597348224
>>597289
Масса же зависит не только от объема, но и от плотности.
Аноним 28/08/20 Птн 10:05:55 597352225
>>597322
а как должна переть энергия, чтобы внутреннее тепло грело планету без внешнего источника энергии? или такого слабого, чтобы не только не грело, но светило слегка. как солнце для плутона, например
Аноним 28/08/20 Птн 10:11:27 597353226
Аноним 28/08/20 Птн 14:20:16 597390227
>>597352
Хорошо должна. Но разогрев из-за ядерного распада вполне себе значимый источник энергии. Не уверен, что осмия-186 хватит, все же период полураспада ну ОСНЕ большой. С другой стороны, если прям всю планету из него слепить, то может его будет настолько дохуя, что и хватит. Считать надо.
Аноним 28/08/20 Птн 14:22:36 597393228
Аноним 28/08/20 Птн 14:22:54 597394229
>>597348
Но ведь зависит. Чтобы гравитация оставалась постоянной "при увеличении радиуса", нужно чтобы плотность (причем всей планеты в среднем, а не "свежедобавленного вещества) падала обратно пропорционально радиусу.
Аноним 28/08/20 Птн 14:50:10 597400230
>>597390
>Считать надо.
V≈1.56×1010km3
m = 352,716,000,000,000,000,000kg = 3.53x1017kg = 3.53x1020g
mm = 190,23 g/mol
1.12e+42 атомов осмия
Полураспад альфа - 5МэВ.
Период полураспада - 2х1015 лет.
5е+41 атомов осмия за 2х1015 лет выделят по 5 МэВ энергии.
25е+41 МэВ = 16e+22 джоуля
Примерно 10 мегаджоулей энергии в год.

Площадь поверхности 30 миллионов квадратных километров. Реликтовое излучение приносит 1мкВт/м3
Реликтовое излучение сообщит 30 мегаджоулей в год.

Выбери другой изотоп/атом.
Аноним 28/08/20 Птн 14:53:53 597401231
>>597400
С другим хуево, там по условию ОП осмий хотел. Все остальные слишком уж радиоактивные. Значит надо приливными силами как-то.
Аноним 28/08/20 Птн 14:56:06 597402232
>>597401
Ну тащемта мы про тектонику говорили, так что сугрев нужен, можно что-то и другое кроме осмия, тяжелых атомов еще немало.
Инб4 планета из оганессона распидорашивающаяся что твоя сверхновая в первую микросекунду.
Аноним 28/08/20 Птн 14:58:18 597403233
>>597402
>можно что-то и другое кроме осмия
Вообще, конечно, можно, но по условию нельзя:
>какого радиуса будет планета из чистого осмия
Аноним 28/08/20 Птн 15:00:52 597404234
>>597403
Я разрешаю не из осмия, я просто выбрал что поплотнее.
>>597290-анон
Мы уже посчитали что надо, теперь другой вопрос возник же - чтоб ОСНЕ плотная планета, но еще и с тектоникой. Хтонично будет выглядеть.
Бонусом было бы низкое альбедо, но плотные пепяки обычно металлы и будут сиять.
Аноним 28/08/20 Птн 15:03:17 597405235
>>597404
Ну тогда сыпани классического калия-40. У него период полураспада в миллион раз меньше, но все равно это больше миллиарда лет.
Аноним 28/08/20 Птн 15:06:22 597407236
>>597405
Меня смущат что я взял энергию альфа-распада в 5МэВ, она всегда такая что ли? Это и будет тепловая энергия разогрева недр?

Тока это, КАЛий легкий дохуя же, задумывалась изначально мелкая зловещая планета из плотного металла чтоб ухбля, пот на лбу выступал.

Хтонические планеты такими могут быть, или там камня налипает дохуя? Кто-то кукарекал про алмазные дожди на горячих юпитерах, может после сдува атмосферы остаться поля алмазов?
Аноним 28/08/20 Птн 15:12:12 597408237
>>597407
>Меня смущат что я взял энергию альфа-распада в 5МэВ, она всегда такая что ли?
В душе не ебу, я твоим рассчетам поверил наслово.
>Это и будет тепловая энергия разогрева недр?
По идее да, вряд ли что-то из продуктов деления сможет пробиться наружу через сотни километров осмия и просто так улететь. Все должно в тепло перейти.
>Тока это, КАЛий легкий дохуя же, задумывалась изначально мелкая зловещая планета из плотного металла чтоб ухбля, пот на лбу выступал.
Тогда уран-238.
>Хтонические планеты такими могут быть, или там камня налипает дохуя?
Да никто не знает, есть только более-менее обоснованные теории. Но что естественная планета из чистого осмия (даже если с небольшими примесями урана или калия) невозможна, тут я готов анус поставить.
Аноним 28/08/20 Птн 15:16:39 597409238
image.png 265Кб, 480x360
480x360
>>597408
>тут я готов анус поставить
Это сайфай уже больше и эксперименты в уме "а что если". Из кротов планет тоже нет (вроде бы), а обсудить интересно.
Аноним 28/08/20 Птн 15:18:12 597410239
>>597408
Мы может не с той стороны подошли. Я выбрал самый тяжелый элемент, а ведь может быть какие-нибудь молекулы/сплавы могут быть плотнее, нет?
инб4 нейтрониум
Аноним 28/08/20 Птн 15:20:26 597411240
>>597409
Ну если так, мои рассуждения такие, что в любой металлической планете будут большие (грубо говоря, половина) примеси всяких кислородов, сер и прочих неметаллов. Химически все это отлично связывается в оксиды и всякие соли, а разделение требует нечеловеческих усилий. Какому-нибудь солнечному ветру такое не под силу, он только атмосферу может сдуть, ну так она и без того с металлами не связана.
Аноним 28/08/20 Птн 15:22:50 597412241
>>597411
А что если был инцидент сопоставимый с тем, что гипотетически произошел с Психеей? Была ОСНЕ большая планета, все тяжелое говно утонуло, оксиды-хуиды вытеснились чистыми расплавами, а потом кто-то по ней уебал, или рядом нейтронки слились окропив атмосферу и камень свежей струей урины тяжелых металлов и сдув все что легче железа?
Аноним 28/08/20 Птн 15:25:22 597414242
>>597410
Из вики:
>Самые плотные вещества при стандартных условиях — металлы платиновой группы (рутений, родий, палладий, осмий, иридий, платина). Имеют плотность 21400—22700 кг/м³.
Скорее всего имеются в виду вещества вообще, а не только простые. Ну и чисто логически, плотность определяется массой атомов (масса растет куда быстрее объема, поэтому плотность нарастает) и межатомными расстояниями. Для кристаллов, например, типом решетки. Даже если там бывает что-то плотнее чистого осмия, то это наверняка какой-то сплав все тех же металлов. Явно уж не оксиды какие-нибудь, прости господи.
Аноним 28/08/20 Птн 15:28:30 597415243
>>597414
А мы один момент для сугрева забыли.
Планета не будет представлять из себя однообразный шар с одинаковой плотностью и давлением. В центре скукожит атомы поплотнее.
Заспавненный равномерный шар из осмия осядет на километров 10 сразу разогрев при этом внутренности.
Аноним 28/08/20 Птн 15:28:40 597417244
>>597412
Ну хз. Но чтобы было такое ядро из осмия, нужно чтобы были соответствующие слои из более легких металлов, которых куда как больше (того же железа должно быть на порядки больше). Что-то исходная планета по массе уже в звезду превращается, такими темпами.
Аноним 28/08/20 Птн 15:39:31 597420245
>>597412
Железо всего в 3.5 раза тяжелее камня. Не думаю, что, то что сдуло камень не сдует железо, и соответственно всю планету целиком. Да и нет особо в космосе катаклизмов способных сдуть камень. А если бы и могли это 1 на 1000 что сдует только камень.
Вот трансмутировать нейтронкой каменную поверхность в металлическую может быть. Но как ты понимаешь это скорее будет какое-то говно из разно-разложившихся элементов, где вся таблица элементов. В итоге это скорее всего превратится в камни только не в силикатные, а какой-нибудь полоний-мышьяк.
Аноним 28/08/20 Птн 15:42:13 597422246
>>597420
>трансмутировать нейтронкой каменную поверхность в металлическую
Я чот сильно сомневаюсь. Если идет такой поток нейтронов, то планету просто испарит нахуй. Ну, то есть, вещество может и трансмутируется, но планету оно уже составлять не будет, как мне кажется.
Аноним 28/08/20 Птн 15:42:42 597423247
>>597394
Тут в соседних постах планету из чистого осмия обсуждают, а ты что-то про плотность несешь.
Аноним 28/08/20 Птн 16:46:00 597440248
Опа, тут как раз про кольца Сатура в ОП-посте.
Какой объект нужно разрушить на орбите Земли, чтобы появились кольца, достаточно видимые с поверхности нашей планеты? А можем ли мы, например, прямо завтра взять, и грамотно направить ядерными взрывами спутники Марса в сторону Земли, чтобы они разрушились и образовали видимые кольца?
Аноним 28/08/20 Птн 17:18:32 597449249
>>597440
Тротила не хватит. Едва ли как-то все наши ядрёны изменят орбиту спутников марса, и уж тем более переместит их к земле.
А если мы говорим о мимо-метеоритов, то их слишком мало, и мы вряд ли их сможем затормозит на орбите.
Кольца в принципе возможно создать, но не при текущем развитии, да и ближайшие сотни лет вряд ли. Нужно кучу материала отбуксовать, а это дохрена энергии. Где эту энергию брать я даже не представляю. Ядерные буксиры должны быть массовым продуктом и скорее всего даже если им хватит тяги, ядерного материала может и не хватить. А в итоге ненужная хрень, лучше бы звёзды покоряли.
Аноним 28/08/20 Птн 17:22:57 597452250
>>597440
>А можем ли мы, например, прямо завтра взять, и грамотно направить ядерными взрывами спутники Марса в сторону Земли, чтобы они разрушились и образовали видимые кольца?
Направить-то можем, только не взлетитулетит. Да и вдвоём Деймос с Фобосом не смогу создать колец, чтобы было К Р А С N В 0. Нужно распидорасить что-то не меньше Плутона.
Тем более кольца будут осыпаться на поверхность, но это уже совсем другая история
Аноним 28/08/20 Птн 17:40:32 597457251
>>597452
>что-то не меньше Плутона
Ты ебанулся, даже у Сатурна кольца весят в триста раз меньше.
Аноним 28/08/20 Птн 17:57:46 597465252
>>597440
>направить ядерными взрывами %хуйнюнейм% к %планетанейм%
Лолнет. Просто для понимания, один лишь Фобос весит с десяток миллионов тонн, а Деймос раз в сто больше.
Аноним 28/08/20 Птн 18:09:18 597468253
>>597465
>Фобос весит с десяток миллионов тонн
Десять триллионов.
Аноним 28/08/20 Птн 18:43:05 597472254
Почему бы не ударить по Луне ядерными ракетами? Глядишь и появится там своя атмосфера.
28/08/20 Птн 18:56:10 597474255
>>597465
>а Деймос раз в сто больше.
Чо несёт? У тебя Деймос тяжелее Фобоса? Фобос в 7 раз тяжелее Деймоса
Аноним 28/08/20 Птн 19:52:37 597486256
Аноним 28/08/20 Птн 23:37:59 597510257
>>597474
Он другую солнечную систему нароллил, видимо.
Аноним 29/08/20 Суб 05:51:04 597556258
>>597468
Кстати, здание МГУ весит около полумиллиона тонн, для сравнения.
Аноним 29/08/20 Суб 08:25:43 597562259
>>597452
>можем ли мы, например, прямо завтра взять
>Направить-то можем
Из какого века капчуешь? Луну заселили? На Марсе город построили?
Аноним 29/08/20 Суб 08:29:37 597564260
ayy lmao in the[...].jpg 25Кб, 500x375
500x375
>>597562
>12020 г.э.
>до сих пор даже не начали колонизировать свою солнечную систему
Аноним 29/08/20 Суб 13:58:16 597601261
>>597562
Всё построили, всё наладили! Правильную тогда Рогозин программу выбрал.
Аноним 29/08/20 Суб 17:04:21 597656262
Допустим совершаю 10-50 запусков на орбиту, чтобы собрать там корабль и ещё 1000 запусков чтобы доставить топливо и окислитель.
Что мне помешает разогнаться на построенном корабле например до 90% скорости света?
Аноним 29/08/20 Суб 17:04:53 597657263
>>597656
удельный импульс
Аноним 29/08/20 Суб 17:06:13 597658264
>>597657
Что удельный импульс?
Выходящие из сопла газы будут толкать ракету пока не кончится топливо и окислитель.
Аноним 29/08/20 Суб 17:06:27 597659265
Аноним 29/08/20 Суб 17:06:51 597660266
Аноним 29/08/20 Суб 18:02:23 597672267
15887887274540.jpg 79Кб, 741x561
741x561
>>597656
Ну допустим у тебя даже не хим. топливо, а ксенон и удельный импульс 15000. Ну или водород и газофазный ЯРД. Пустой корабль весит 1000 тонн, ксенон в нём 100 тыс т., у нас ультра-баки будущего. Выкипанием, выгоранием, охладом и прочим ненужным задротским говном пренебрежём. А разгоняешься ты посреди абсолютного нихуя, так что потерь на не-мгновенность у тебя нет. При таких условиях ты разгонишься аж до целых невероятных сумасшедших 700км/с. Можешь разбить на ступени, ситуация принципиально не изменится.

>>597658
Проблема в том, что уравнение Циолковского имеет экспоненциальный характер. Так что все претензии можешь слать этому злобному старикашке.
Аноним 29/08/20 Суб 18:47:46 597677268
>>597672
Инбифо "что мне помешат долить еще сто миллиардов тонн топлива".
Аноним 29/08/20 Суб 18:50:16 597678269
>>597656
Забиваешь в поисковую строку гугла
V * ln K
Где v-скорость истечения топлива
K-во сколько раз масса корабля с топливо больше массы пустого корабля
и получаешь скорость корабля по истечению топлива
при скорости истечения 5км/с (водород-кислород из супир-трупер двигателя)
и K=1000 000 (ну очень много раз запускали ракету-дозаправщик)

получим 0.00023 скорости света
Аноним 29/08/20 Суб 19:42:39 597680270
>>597672
а если взять хуйню на пылевой плазме или еще хуже?
Аноним 29/08/20 Суб 20:36:47 597684271
>>597677
>что мне помешат долить
Гравитация, лол.
Скажем при УИ 23,5 км/c (2400 секунд) и "волшебном" преобразовании топлива, т.е. если у нас есть только ПН и топливо, а баки и двигатели весят ровно 0, и разгоняем мы ажно целый килограмм ПН,то где-то к 1400 км/c (менее 0,5% с) масса топлива превысит массу Нептуна. У которого скорость убегания как раз те самые 23,5 км/c.
Можно конечно поигратся с цифрами, но фнудаментально это ничего не изменит - пока скорости истечения ограничены сотнями-тысячами км/c, скорости сравнимые со световой, скажем 10% и более, будут требовать таких запасов топилва, что они будут удовлетворять МАС-овскому определению планеты со всеми вытекающими.
Аноним 29/08/20 Суб 20:46:24 597685272
Обладая современными технологиями обнаружения экзопланет, на каком максимальном расстоянии мы смогли бы обнаружить Землю, наблюдая со стороны?
Аноним 29/08/20 Суб 21:02:13 597687273
>>597685
Вопрос слишком расплывчатый, поэтому ответов несколько.
Гравитационным микролинзированием - как минимум с десятков тысяч светолет, но только если повезёт.
Транзитным методом - с тысяч, зато он универсален.
Обнаружить объект размером примерно с Землю и нагадать на кофейной гуще что он в обитаемой зоне данной звезды - сравнительно легко. Понять что там есть кислородная атмосфера, вода, и т.п. - намного сложнее.
Надо понимать, что современные технологии обнаружения развиваются оче быстро, и что первую экзопланету транзитным методом нашли всего 18 лет назад, а массовому поиску едва с десяток лет, по сути.
Аноним 29/08/20 Суб 21:07:16 597688274
>>597687
Разве найдены планеты на таком же расстоянии от звёзд как земля? В основном же горячие Юпитеры, на расстоянии меньше орбиты Меркурия.
Аноним 29/08/20 Суб 21:10:28 597689275
Аноним 29/08/20 Суб 21:11:43 597690276
>>597689
Это гравитационное микролинзирование, без малого 25 тыс световых лет, 4 массы Земли.
Аноним 29/08/20 Суб 21:13:09 597691277
>>597688
В смысле вообще, а не на больших дистанциях от нас? Полным-полно, есть даже совсем близкие кандидаты.
Аноним 29/08/20 Суб 21:16:09 597692278
>>597689
Совсем другие параметры масс и всего прочего. Да и расстояние значительно меньше 1 АЕ.
Аноним 29/08/20 Суб 21:26:53 597694279
>>597692
Ну какие другие? Это всего-то 4 массы. Вполне себе не гигант, обычная суперземля, которых куча. Это был пример далёкого обнаружения суперземли, а не похожести на Землю.
>расстояние значительно меньше 1 АЕ
Это имеет значение лишь для транзитного метода. Разные методы имеют разную чувствительность для разных расстояний. Каким-то желательно поближе к звезде, каким-то (например прямое наблюдение) наоборот подальше.
Аноним 29/08/20 Суб 21:39:34 597697280
>>597687
>зато он универсален
Там тоже надо, чтобы плоскость орбиты экзопланеты была почти параллельна нашему лучу зрения.
Аноним 29/08/20 Суб 22:30:26 597701281
image.png 8Кб, 771x375
771x375
Итак здравствуйте аноны, не знаю куда конкретно написать по данному вопросу поэтому насру здесь.

Время было около 8ч по мск, 48 град северной широты, центральная россия, 29.08.2020 , в небе пролетело 2 пары объектов с интервалом около 3 минут абсолютно беззвучно, дело происходило в городской черте и засветка неба была достаточно велика и при этом была возможность их разглядеть, примерно 3/4 небосвода объекты преодолели секунд за 5-6, нет это были не спутники, так как диаметр свечения был слишком велик для спутников, светимость порядка небольшого метеора в верхних слоях атмосферы, цвет свечения что то типа коричнево-красного, самое интересное это траектория первой пары объектов, пикрил(красными линиями траектория черные точки приблизительный размер объектов как я их видел),

вторую пару я проебланил и увидел только когда она уже подходила к горизонту, и траекторию оных нарисовать не смогу,

При том что один объект первой пары имел синусоидальную проекцию траектории на мои глазищща, он летел паралельно второму,т.е. не отставал

ИТАК ВОПРОС
ЧТО Я МОГ ЛИЦЕЗРЕТЬ?
Аноним 30/08/20 Вск 03:15:20 597708282
>>597701
Не стоит вскрывать...
Аноним 30/08/20 Вск 04:16:31 597712283
this is getting[...].jpg 62Кб, 1280x720
1280x720
>>597701
Ты видел НЛО! Неопознанные летающий объекты!
Скорее всего этими объектами были китайские фонарики летящие на относительно низкой высоте при большом ветре.
Аноним 30/08/20 Вск 05:07:02 597717284
image.png 114Кб, 654x410
654x410
image.png 2083Кб, 1916x959
1916x959
>>597712
Фонарики намного ярче при такой высоте и имеют четкие формы,вечер был безветренным, буквально, а это просто размытое облачко темно красного света
Аноним 30/08/20 Вск 05:13:24 597718285
>>597717
>вечер был безветренным
То показывается погода на поверхности, на 200м не меряют ветра, где они летают.
Аноним 30/08/20 Вск 05:16:01 597719286
>>597718
в любом случае, с той стороны откуда они летели была волга, за волгой безжизненные выжженные пустоши, я видел фанарики китайские и низко и высоко и могу их отличить от неебической летающей хуйни, поэтому с уверенностью отвергаю предположение на правах наблюдателя данной неебической летающей хуйни
Аноним 30/08/20 Вск 05:19:00 597720287
image.png 3341Кб, 2000x1333
2000x1333
>>597719
в интернете отыскал релевантную фотографию, но там видать выдержка большая, и объект длинноватый и яркий на зуме однако цветность передает отлично
Аноним 30/08/20 Вск 05:24:23 597721288
сука вот блять заказал бы я ебаную дифракционную решетку хоть блять бы спектр был чо там горело если горело вообще
Аноним 30/08/20 Вск 05:27:21 597722289
>>597720
На этой фотографии самолет.
Аноним 30/08/20 Вск 05:35:23 597723290
>>597722
перечитай первый мой пост
погляди на траекторию
прочитай про бесшумность
+ проблесковые маячки на гражданский и частных рейсах мигают с переодичностью
+самолет гражданский не развивает такую скорость и не имеет такой размах крыльев в угловом размере на достаточной высоте чтобы не слышать его
Аноним 30/08/20 Вск 05:38:22 597724291
первая мысль в голове была когда я это увидел, тоже была про самолет, однако ни подтвердить , ни опровергнуть , у меня не получилось , потому что думалка не сработала моментально и я не сообразил обратить внимание пропадает ли свет звезд в пространстве между источниками света (закрывает ли брюхом самолет/нëх их)
Аноним 30/08/20 Вск 05:40:23 597726292
>>597723
Значит ты никогда в жизни самолеты не видел.
Это самолет.
Дело закрыто.
Аноним 30/08/20 Вск 05:41:15 597728293
ну и чтобы внести какую-то ясность в расстояние между источниками света первой пары могу сказать что в наиболее дальней друг от друга позиции растояние между ними было равно приблизительно в раза полтора-два больше диска луны
Аноним 30/08/20 Вск 05:42:00 597729294
>>597726
щас пойду и чекну рейсы в тот промежуток времени
Аноним 30/08/20 Вск 05:45:15 597730295
с телефона фр24 неудобно пользоваться, посему попрошу тебя просто собрать свои доводы воедино
Аноним 30/08/20 Вск 05:56:01 597732296
скачал фр посмотрел за вчера с трехчасовым запасом вперед-назад
ничего зафиксированного фдайтрадаром над местом наблюдение не летало
Аноним 30/08/20 Вск 06:14:34 597734297
Стикер 191Кб, 500x500
500x500
>>597729
Малая и военная авиация на флайтрадаре не отображается.
Аноним 30/08/20 Вск 11:52:33 597759298
Я один никогда не воспринимал солнце как звезду?
Аноним 30/08/20 Вск 11:54:21 597760299
>>597759
Когда ты задаешь вопрос "я один..." - ответ в 99.999999% случаев "нет".
Подавляющее большинство людей не воспринимало Солнце за звезду, да и сейчас полно таких.
Аноним 30/08/20 Вск 11:54:36 597761300
Аноним 30/08/20 Вск 11:56:59 597762301
>>597760
Я просто всегда читал про нейтронные звезды, чёрные дыры, а потом осознал что мы крутимся вокруг ёбаной звезды, изучай не хочу, хочу изучать говно в 99999млн км

Как же хочется солнечный телескоп
Аноним 30/08/20 Вск 12:05:06 597763302
>>597762
Кто "не хочу"? Солнце - самая изученная звезда во вселенной. Солнечных обсерваторий жопой жуй, постоянное наблюдение за всеми видами активности.
Аноним 30/08/20 Вск 13:06:26 597771303
>>597761
скорость малая у спутников на видео, те же преодолели 3/4 небосвода за секунд 6
Аноним 30/08/20 Вск 13:12:49 597772304
>>597771
Так это не спутники.
Аноним 30/08/20 Вск 13:14:10 597773305
>>597771
да и траектория очень странная у одного из объектов
Аноним 30/08/20 Вск 13:36:31 597774306
>>597771
Отражение фонарика может развивать сколь угодно высокую скорость.
Аноним 30/08/20 Вск 14:49:22 597779307
Кароч придумал новый вид корабля. Выводим на орбиту обычной ракетой. Дальше ускоряем ионными двигателями. Рабочее тело берём прямо из космоса. Энергию от солнечных батарей или от ядерного реактора. Таким образом постепенно поднимаем орбиту. Можно ускоряться очень долго вплоть до 20% скорости света.
Аноним 30/08/20 Вск 14:53:19 597783308
>>597774
отражение обо что?
Аноним 30/08/20 Вск 15:13:25 597788309
Поясните за пардокс атмосферы.
Есть одна Земля - о её атмосфере, жидкой воде и биомах с форой и фаунов, достоверно известно.
Есть Титан - у него есть атмосфера более плотная, чем земная, хотя Титян и меньше, и легче
Есть Свинера - у неё масса и размеры боле менее как Земля, но атмосфера плотнее и толще.
Есть Марс - с рябой и кривой, но атмсоферой имени блохастой собаки.
Мы знаем, что жидкость на поверхности регулируется атмосферой, и что при определённом наборе масс-объёма, жидость не будет выпариваться в космос, а останется на планете, ибо пойдёт в облака и в осадок, вернувшись к истокам. Атмосефра же защищает от межзвёздной и местносолнечной радиации и особенно ветра, а следовательно и от радиолиза. Но всё это только при наличии достаточно толстой атмосферы.
Но почему так?
Получается, налей мы на Титан мало атмосферы, он был был лоховским Ганимедом. Налей мы на Марс дохуя атмосферы, там и вода была бы жидкая, и яблони бы цвели. Да даже на земной Луне если вообще дохуя атмосферы налить, можно получить атмофсеру, который и лунной гравитации как раз,и солнечный ветер не страшен и земная гравитация.
Получается, существования атмосферы поддерживается... существованием атмосферы?
Аноним 30/08/20 Вск 15:28:41 597792310
>>597779
>новый вид
Роберт Бассард из-за тебя снижает крутящий момент Земли.
Аноним 30/08/20 Вск 15:32:33 597793311
>>597760
>Когда ты задаешь вопрос "я один..." - ответ в 99.999999% случаев "нет".
Вот, сука, двачую. Меня одного такое бесит?
Аноним 30/08/20 Вск 16:21:03 597804312
Стикер 191Кб, 225x225
225x225
Аноним 30/08/20 Вск 17:00:29 597812313
>>597783
Об лёгкую облачную дымку. О провода. О много что.
Алсо насколько я помню у вас там на волге какой-то видеоблохер своим фонариком всех слепит и облака и волгу и т.д.
Аноним 30/08/20 Вск 19:18:15 597841314
>>597788
Тут важен баланс массы планеты, состава атмосферы, и ее температуры. Чем толще атмосфера, тем (при некоторых условиях) выше температура планеты и самой атмосферы. Чем выше температура- тем ближе скорость движения молекул газа ко второй космической, и соответственно тем быстрее атмосфера съебывается.
Аноним 30/08/20 Вск 19:21:36 597842315
>>597841
Еще ОСНЕ важна геологическая активность. Если вулканы постоянно подпердывают дополнительным газком, то атмосфера даже при довольно быстром рассеивании сможет оставаться плотной.
Аноним 30/08/20 Вск 19:26:01 597845316
Аноним 30/08/20 Вск 20:19:59 597853317
>>597812
эт дурачки с прожекторами вроде цирк или еще что то, на облаках часто бывает да, но тогда и облаков не было, звезды прекрсно были видны
Аноним 30/08/20 Вск 20:25:46 597857318
>>597788
Атмосфера Земли биогенная, если б не жизнь она была бы совсем другой. И тащемта была - в ранние моменты её существования, да и после кислородной катастрофы и давление и состав долго были непохожи на то что сейчас.
Аноним 30/08/20 Вск 20:27:39 597858319
>>597788
>Получается, существования атмосферы поддерживается... существованием атмосферы?
Ну да. Что тебя удивляет? Титан перетянул на себя дохуя при формировании, поэтому он такой. Венера ещё больше, поэтому она такая. А уж газовые гиганты перетянули охуеть сколько. Выдувай - не хочу, солнце раньше лопнет.
Аноним 30/08/20 Вск 20:59:22 597860320
>>597299
Бимп. Ну что же вы, анонимные ученые?
Аноним 30/08/20 Вск 21:40:53 597865321
>>597860
Взял бы да погуглил.

Забиваешь в гугл Ganymede moment of inertia, шаг 1: https://en.wikipedia.org/wiki/Moment_of_inertia_factor, шаг 2: https://en.wikipedia.org/wiki/Darwin%E2%80%93Radau_equation

Момент инерции для тел, находящихся в гидростатическом равновесии, т.е. приобретшим форму эллипсоида вращения под воздействием собственной гравитации и центробежных сил, можно вывести из соотношения между массой, скоростью вращения и полярным сжатием.

Утрированный пример — если бы 99,9% массы Ганимеда было сосредоточена в точке в его центре, а остальная планета состояла из сахарной ваты, то никакого полярного сжатия бы вообще не было — гравитационное поле было бы практически идеально сферическим. И наоборот, если бы он был полностью однородным, то полярное сжатие бы полностью соответствовало теоретическому для однородной несжимаемой жидкости.

Зная, где именно Ганимед находится между двумя этими крайностями, можно примерно так почувствовать, что у него внутри.
Аноним 30/08/20 Вск 22:10:32 597874322
Аноним 31/08/20 Пнд 02:11:20 597913323
Сколько сантиметров слоя атмосферы с юпитера надо снять, чтобы покрыть все планеты, спутники, карликовые планеты солнечной системы атмосферой с плотностью одной земной?
Аноним 31/08/20 Пнд 02:23:03 597921324
>>597913
Большая часть всего этого зоопарка атмосферу с плотностью земной неспособна удержать в принципе, хоть ты весь юпитер туда надуй.
Аноним 31/08/20 Пнд 02:56:44 597945325
>>597845
Что значит "кроме Марса"?
На марсе тоже будут свои часы.
Предполагается что 37 минут будут идти как двадцать пятый час.
После 23:59:59 пойдет 24:00:00 и так до 24:37:00, после чего 0:00:00
Аноним 31/08/20 Пнд 11:27:28 597970326
Почему СССР построил космодром в Казахстане, а не в более южной Туркмении?
Аноним 31/08/20 Пнд 11:33:46 597972327
>>597970
Надо тысячу с лишним километров своей территории для зон падения ступеней. В китай бы падало, что не айс.
Мне так кажется.
Аноним 31/08/20 Пнд 12:27:33 597982328
Придумал как добывать газ. Берём очень большой пылесос и засасываем газовые гиганты.
Ну как вам идея?
Аноним 31/08/20 Пнд 12:28:06 597983329
Аноним 31/08/20 Пнд 12:28:11 597984330
>>597982
Годно. Приступай.
Аноним 31/08/20 Пнд 12:32:09 597986331
image.png 74Кб, 304x166
304x166
Возможно ли построить город на колцах сатурна? Там ведь много астеройдов по размерам как раз подходящих для дома, они близко и их можно соединить дорожками. Представьте как там будет красиво эх... вместо солнца будет сатурн. Готовы купить там домик?
Аноним 31/08/20 Пнд 12:35:08 597987332
>>597986
>Возможно ли построить город на колцах сатурна?
Нет.
Аноним 31/08/20 Пнд 12:37:21 597989333
Аноним 31/08/20 Пнд 12:41:14 597991334
Стикер 191Кб, 500x500
500x500
>>597989
Что и ожидалось от ГАЗОВОГО ГИГАНТА.
Аноним 31/08/20 Пнд 12:42:07 597992335
>>597991
Что сказать-то хотел?
Аноним 31/08/20 Пнд 13:46:01 598013336
>>597986
Если населить его войнами, то почему нет?
Аноним 31/08/20 Пнд 13:49:17 598014337
>>598013
Если эти войны будут людьми, то они будут легко умирать, надо им поддержку, СЖО. Лучше если они были бы андройдами.
Аноним 31/08/20 Пнд 14:00:13 598016338
image.png 137Кб, 218x218
218x218
Аноним 31/08/20 Пнд 14:01:03 598017339
Стикер 63Кб, 512x451
512x451
>>598016
Кажется, этот ГАЗОВЫЙ ГИГАНТ ВОСПЛАМЕНИЛСЯ.
Аноним 31/08/20 Пнд 15:07:21 598025340
Суп хвощ. А если построить или выкопать на луне охуенных размеров параболическое сопло, а потом подрывать в нем термоядерные боньбы, это ведь будет создавать тягу? Можно таким образом разгонять Луну?
Аноним 31/08/20 Пнд 15:17:26 598026341
>>598025
Да.
Это будет создавать тягу.
Можно таким образом разгонять луну.
Более того, выкапывать ничего и не надо, первые взрывы сами сделают это параболическое зеркало.
Аноним 31/08/20 Пнд 15:55:34 598028342
>>598025
Тягу можно создавать даже высирая говно и кидаясь им в небо на скорости больше второй космической. Вопрос в эффективности.
Аноним 31/08/20 Пнд 17:33:46 598031343
>>597921
Предположим, смогут удержать.
Аноним 31/08/20 Пнд 17:40:24 598037344
>>598031
Так а вопрос смысла не имеет. Максимальная плотность атмосферы зависит от гравитации данного тела, поэтому условия из задачи невыполнимы в принципе. Кроме того, от гравитации зависит и то, сколько кг атмосферы нужно надуть, чтобы достичь данной плотности (в тех случаях, когда это вообще возможно), поэтому так вот просто "меняя" гравитацию, ты меняешь ответ.
Аноним 31/08/20 Пнд 18:17:53 598047345
А что вызывает ядерный распад? Что является именно причиной того что данный изотоп в данный момент распался?
Аноним 31/08/20 Пнд 18:35:03 598054346
image.png 595Кб, 2624x1243
2624x1243
Поясните про квантовую запутанность.
Почему типа измерение одной штуки как-то влияет на другую?
Они же одинаковые вышли изначально, они и будут одинаковые.
И то что мы не знали их состояния лишь говорит про то что мы их не знали, а не про то что они квантовые-хуянтовые.
Вообще вся эта квантовая хуйня кажется полным бредом и наебаловом.
Вот ты посмотрел и типа изменил что-то. А не посмотрев ты результат не узнаешь. А узнав результат ты на оригинал не влияешь.
Хуета какая-то раздутая на ровном месте, так мир не работает.
Аноним 31/08/20 Пнд 18:55:44 598060347
>>598047
В ядрах атомов действует сильное ядерное взаимодействие, которое притягивает частицы. Но против него действует электромагнитное отталкивание протонов, которые одинаково заряжены. Электромагнитное взаимодействие слабее, но не так быстро ослабевает с расстоянием.

Некоторые изотопы находятся "на грани": в общем-то ядро собрано в кучку, но случайные колебания частиц могут нарушить баланс, когда частицы разлетаются чуть дальше чем надо - и пиздец, электромагнитное отталкивание пересиливает, все разваливается.

Ну это так, упрощенно, конечно. На самом деле там еще и слабое ядерное замешано, объясняющее бета-распад, например. Да еще и протоны-нейтроны не элементарные частицы, а состоят из кварков, да еще и квантовые эффекты. Но в целом как-то так.
Аноним 31/08/20 Пнд 19:47:04 598065348
>>598054
Грубо говоря это связано с волновыми свойствами частиц. Когда мы измеряем волну мы определяем где находится корпускула. Когда взаимодействуют 2 волны и мы не можем сказать, где находится корпускула в облаке вероятности у нас получается 2 связанные частицы. И когда мы определяем где находится одна из провзаимодействующих частиц, мы можем отмотать время назад и определить где находится вторая частица через первую.
Аноним 31/08/20 Пнд 20:29:31 598072349
>>598037
Тут ещё смотря из чего атмосфера. Одно дело водород или гелий, и совсем другое какой-нибудь ксеон или радон.
Аноним 31/08/20 Пнд 20:31:54 598075350
>>598072
Да. Там много факторов. Температура вот еще, горячий газ менее плотен. Но так или иначе вопрос бессмысленен, потому что желаемые условия недостижимы.
Аноним 31/08/20 Пнд 20:32:44 598076351
>>598054
>так мир не работает
А как он работает? Желательно с пруфами.
Аноним 31/08/20 Пнд 23:06:19 598101352
1280px-Bell.svg.png 85Кб, 1280x1097
1280x1097
>>598054
О, еще один познакомился с удивительным миром квантовой физики.

>И то что мы не знали их состояния лишь говорит про то что мы их не знали, а не про то что они квантовые-хуянтовые.
А теперь почитай про ЭПР-парадокс, про Джона Стюарта Белла и про его неравенства.

Оказалось, что можно поставить такой эксперимент, который дал бы разный результат в случаях:
1) Если бы у запутанных частиц было конкретное изначальное состояние, просто неизвестное нам — так называемые "скрытые переменные".
2) Если бы никакого начального состояния не было, частицы действительно находились бы в смешанном квантовом состоянии, а конкретные величины появлялись только при измерении, причем одномоментно у обоих запутанных частиц сразу.

Поставили эксперимент, оказалось, что верен вариант 2, состояние появляется только в момент разрушения запутанности. Скрытых переменных нет, локального реализма нет, квантовая физика это и есть реальность, а не просто удобный способ описания нашего незнания.

То, что выросшим на классической механике людям и ОТО, и квантовая физика кажутся каким-то бредом, никого не волнует, реальность под их хотелки подстраиваться не собирается.
Аноним 01/09/20 Втр 00:30:05 598126353
>>598054
>Вообще вся эта квантовая хуйня кажется полным бредом и наебаловом.
>Вот ты посмотрел и типа изменил что-то. А не посмотрев ты результат не узнаешь. А узнав результат ты на оригинал не влияешь.
Наблюдение невозможно без взаимодействия с наблюдателем, при помощи какой-либо из фундаментальных сил. Собственно, под наблюдением и понимается взаимодействие. Поэтому наблюдение влияет на квантовую систему.
Аноним 01/09/20 Втр 06:31:12 598158354
>>598101
Как оказалось? Бред полный, они просто не знали результат, никак измерение на другие вещи не может влиять, хватит дичь пороть, это не имеет никакого смысла.
Аноним 01/09/20 Втр 08:12:45 598168355
>>598158
Аноним, это тред тупых вопросов, а не тупых утверждений. Если ты и так всё сам знаешь, зачем тебе анонимные учёные мирового уровня?
Аноним 01/09/20 Втр 08:13:34 598169356
>>598168
Чтоб пояснили какого хуя придумали такую ерунду вместо того чтобы честно признать "мы не знаем".
Аноним 01/09/20 Втр 08:18:39 598170357
>>598169
Тебе уже пояснили, почему наблюдение влияет на квантовую систему, максимально простым способом. Или ты видишь это, или продолжаешь срать своим говном.
Аноним 01/09/20 Втр 08:20:56 598171358
>>598170
Не пояснили.
"Поставили эксперимент и оказалось" это пояснение уровня яскозал. Нихуя не показано что эта бридятина имеет смысл.
Аноним 01/09/20 Втр 08:39:07 598172359
Аноним 01/09/20 Втр 08:48:30 598174360
>>598172
Да это понятно, непонятно с хуя ли решили что другая квантово запутанная частица вдруг меняется, а не сразу изначально такими параметрами обладала.
Аноним 01/09/20 Втр 15:29:53 598246361
>>598174
Потому что никаких параметров не существовало до измерения.
Аноним 01/09/20 Втр 15:32:37 598248362
>>598246
Тыскозал?
У всех параметры существуют, а у этих с какого-то беспруфного вскукарека оказывается не существует, ага щас.
Аноним 01/09/20 Втр 22:36:06 598300363
>>597282
Юпидер + Сатурн справа налево
Аноним 01/09/20 Втр 23:58:06 598309364
Какой максимальный размер и массу для наглядности – в землях может иметь спутник газового гиганта?
Аноним 02/09/20 Срд 01:59:43 598318365
>>598309
Не больше самого гиганта. Не ближе предела роша.
Аноним 02/09/20 Срд 02:02:45 598319366
>>598318
И еще внутри сферы Хилла. Чем ближе гигант к своей звезде, тем она меньше.
Аноним 02/09/20 Срд 12:27:57 598376367
>>598318
>Не больше самого гиганта. Не ближе предела роша.
То есть у того же юпитера может быть спутником юпитер поменьше, диаметром и массой 0,9 от юпитера?
Аноним 02/09/20 Срд 12:28:55 598377368
>>598376
Это будет двойная планета, а не спутник.
Аноним 02/09/20 Срд 12:47:41 598382369
>>598377
Будто есть большая разница. Вон тот же Плутон-Харон - вполне себе двойная система, при их разнице в 8 раз по массе барицентр находится над поверхностью Плутона. Тем не менее Харон считается спутником Плутона. Кто меньше - тот того и спутник.
Аноним 02/09/20 Срд 12:52:58 598385370
>>598382
>Будто есть большая разница.
Номенклатурно есть. Это уже не луна.
>Вон тот же Плутон-Харон - вполне себе двойная система
Да. И никто из них не является планетой.
>Тем не менее Харон считается спутником Плутона.
А Плутон не является планетой.
Аноним 02/09/20 Срд 13:01:26 598390371
>>598385
Да какая разница-то? Ну будет у тебя Нептун вокруг Юпитера вращаться например, получится то же самое - одновременно и Нептун-спутник-Юпа, и двойная планета Юпитер-Нептун. Если конечно IAU не придумает каких-нибудь охуенных исключений снова. Всё это вопрос номенклатуры и не стоит выеденного яйца, природа не знает этих категорий, у неё это просто сорта крутящихся по разным траекториям говен.
Аноним 02/09/20 Срд 13:03:10 598391372
>>598390
Хотя ладно, Нептун слишком хилый для Юпа, пусть вон Сатурн обращается, у них меньше разница.
Аноним 02/09/20 Срд 13:04:42 598393373
>>598390
>>598391
Все уже придумано. Сатурн вокруг юпитера будет двойной планетой, а не луной.
Почитай определения, все расписано.
Аноним 02/09/20 Срд 14:26:07 598406374
>>598377
>Это будет двойная планета, а не спутник.
Про максимальные размеры и массу спутника мне так и не написали, только то, что спутник должен быть чуть меньше юпитера.
Аноним 02/09/20 Срд 15:27:04 598414375
>>598385
Всё же я настаиваю, что плутон всё же не планета, а карликовая планета, но планета же пусть и карликовая.
Аноним 02/09/20 Срд 15:42:55 598423376
Io.png 544Кб, 813x787
813x787
Насколько большой может быть каменная планета с твердой поверхностью? Насколько там будет велика гравитация? Может ли такая планета иметь Землю в качестве спутника, а не двойной планеты?
Аноним 02/09/20 Срд 16:34:02 598438377
>>598423
>в качестве спутника, а не двойной планеты
Нет там четких критериев. А если и назначат, то все это так или иначе довольно произвольно. Барицентр? Ну так Луна, например, удаляется, барицентр, соответсвенно, тоже.

Вот етот >>598390 правильно сказал.
Аноним 02/09/20 Срд 17:09:37 598448378
>>598423
Газовые гиганты бывают не крупнее Юпитера, а каменистые могут быть чуть больше земли, из-за сжатия от гравитации.
Аноним 02/09/20 Срд 18:45:05 598473379
>>598423
При приближении примерно к 10 массам Земли каменная планета при формировании натягивает на себя кучу газа и становится ледяным/газовым гигантом наподобие Урана/Нептуна. Если плотность как у Земли, то и гравитация соответствующая будет.

>>598448
>Газовые гиганты бывают не крупнее Юпитера
На самом деле примерно 13 Юпитеров - верхний порог, после которого зажигается реакция и оно становится коричневым карликом.
Аноним 02/09/20 Срд 18:53:53 598478380
>>598473
Возможен ли ледяной гигант с твердой поверхностью? По которой можно ходить?
Интересна планета, которая могла бы быть вдвое больше Земли в диаметре, но с низкой плотностью и примерно равной гравитацией.
Аноним 02/09/20 Срд 19:00:04 598482381
>>598478
Нет, слишком дохуя газа, поэтому четкого фазового перехода нема. Хотя может при определенных условиях и возможны какие-то граничные фазы, при которых относительно резкий градиент киселя.
Аноним 02/09/20 Срд 20:23:51 598524382
Если бы земля была спутником или парной планетой, к какой-либо другой планете, влияло бы хоть как-то на землю, то из чего она состоит при прочих равных?
Аноним 02/09/20 Срд 21:06:32 598543383
15217261527960.jpg 44Кб, 720x720
720x720
>>598524
Это GPT-3? Я не могу распарсить этот коммент своей нейросетью.
Аноним 02/09/20 Срд 22:22:24 598576384
>>598543
Что такое распарсить?
Аноним 02/09/20 Срд 22:28:24 598578385
>>598576
Так шизик свои таблетки называет, которые он забыл принять
Аноним 02/09/20 Срд 23:46:13 598585386
>>598473
>>Газовые гиганты бывают не крупнее Юпитера
>На самом деле примерно 13 Юпитеров - верхний порог
Он, наверное, имел в виду радиус. Там даже если масса растет, радиус остается примерно как у Юпитера (ну чуток больше только) за счет сжатия.
Аноним 02/09/20 Срд 23:50:57 598586387
изображение.png 623Кб, 584x1095
584x1095
Аноним 02/09/20 Срд 23:53:01 598587388
изображение.png 340Кб, 1280x720
1280x720
>>598585
И вот еще иллюстрация
Аноним 02/09/20 Срд 23:53:08 598588389
>>598586
Не вижу ничего про размер планеты, только про массу.
Аноним 03/09/20 Чтв 00:10:05 598592390
Сколько пиропатронов в союзе? Какие есть альтернативные способы помимо пиропатрона?
Аноним 03/09/20 Чтв 00:25:12 598594391
>>598592
Кроме пироболта защёлку или замок можно разомкнуть магнитом или пережигаемой ниткой (последний способ используют в основном для фиксации сложенных солнечных батарей). В принципе все эти способы используют в тех или иных случаях для размыкания силовых конструкций.
Аноним 03/09/20 Чтв 00:48:16 598599392
>>598406
Тебе надо прочитать определение планеты.

>>598414
Она не является тем, что называется "планета".
Аноним 03/09/20 Чтв 00:52:31 598600393
>>598438
Вам номенклатуру соблюдать или шашечки?
Аноним 03/09/20 Чтв 00:56:03 598601394
>>598543
Не переживай, я получил достаточно увечий и черепно-мозговых травм чтобы распарсить.

>>598524
Скорее нет, чем да. Все, из чего состоят планеты создано из примерно одного и того же.
С Меркурия сдуло всю пародию на атмосферу, а жиробусы врое Юпитера будучи поотдаль удержали газики. А в целом у всех дофига железа, кремния и кислорода среди прочих. Именно планетообразование, коллизии и дальнейшая жизнь влияет на то что они из себя станут представлять.
Аноним 03/09/20 Чтв 02:29:52 598603395
Можно ли использовать МКС как межпланетную станцию?

Можно ли переоборудовать Шаттл как корабль для инопланетных колонистов?
Аноним 03/09/20 Чтв 02:45:20 598604396
>>598599
"карликовая акула" не является "акулой".
Аноним 03/09/20 Чтв 03:04:09 598605397
>>598604
Ну, правильно. А карлан - не человек.
Аноним 03/09/20 Чтв 04:27:16 598609398
>>598603
> Можно ли использовать МКС как межпланетную станцию?
Нет, она в принципе не предназначена для использования вне низкой орбиты Земли. Рассчитана на регулярное снабжение, связь ближняя, нет должной радзащиты ни у электроники, ни у жилых объёмов, нечем пулять такую массу на межпланетную траекторию и тормозить, не рассчитана на сколько-нибудь заметную тягу (банально батареи с радиаторами сложатся нахрен, да и сама станция), площадь батарей рассчитана на солнечную постоянную у Земли, и т.д. и т.п., тысячи причин.
>Можно ли переоборудовать Шаттл как корабль для инопланетных колонистов?
Шаттл это тоже корабль исключительно для низких орбит. У него было ещё больше ограничений, потому что его пилили для всего и сразу, и всё он делал плохо. Автономности у него нет. У него и термодинамические ограничения были, он в определенных случаях нуждался в земной тени, чтобы охладиться. Войти в атмосферу Земли с межпланетной траектории он тоже не может.
Аноним 03/09/20 Чтв 05:55:30 598614399
Если бы частная компания воссоздала шаттл, как у Наса почти в точности с такими же ттх, они смогли бы получать прибыль и иметь спрос и нишу на рынке сегодня?
Аноним 03/09/20 Чтв 08:14:35 598624400
>>598614
Нет конечно. Даже отбросив в сторону оверинженеренность самого шаттла - у пилотируемой ебалайки в коммерции попросту нет особых задач. Таскать лысых обезьян на каждый пук вообще рискованно и дорого, а уж выход в открытый космос это вообще туши свет, проще робота послать.
Аноним 03/09/20 Чтв 09:50:26 598640401
>>598604
Нерелевантно.
Карликовая планета не планета. Факт. Все, съебись теперь, даун тупорылый, говно ебаное.
Аноним 03/09/20 Чтв 09:51:39 598641402
image.png 164Кб, 623x343
623x343
Аноним 03/09/20 Чтв 10:25:17 598646403
X-33larcbig.jpg 19Кб, 600x480
600x480
Почему бы не сделать первую ступень ракеты в виде самолета с несущим фюзеляжем? Не нужно ебаться с ховерслэмом и запасать для него топливо, не нужно таскать бесполезные выдвижные крылья, просто взлетел, выкинул вторую ступень, сел даунрейндж как самолет, там кропаль заправился и вернулся обратно на космодром. Или можно даже целую цепочку из космодромов сделать, ракета взлетает, первая ступень садится на следующем космодроме, там на нее надевают вторую ступень, запускают, и так до последнего космодрома, где уже производят ТО и отвозят на первый космодром.
Аноним 03/09/20 Чтв 10:27:11 598647404
Аноним 03/09/20 Чтв 10:29:42 598649405
Аноним 03/09/20 Чтв 10:31:24 598650406
Не надоело быть такими задротами? Еще и деньги наверное на эту хуиту тратите. Сходили бы с друзьями забухали, попиздились с кем нибудь после клуба, потом к машке давалке. Вот она жизнь, а нет там где то под хуйпитером и урАНУСом
Аноним 03/09/20 Чтв 11:04:14 598654407
>>598649
Не слушаю его. Он до сих пор активен что ли, зачем вспомнил?
Аноним 03/09/20 Чтв 15:56:00 598710408
Скорость вращения Земли на экваторе немного меньше 500 м/с. Допустим, увеличив трение об эфир магически мы останавливаем Землю за неделю, для этого должно хватить ускорения в 0,001 м/с^2, даже меньше. То есть, само по себе это должно быть охуенно мало, никто и не почувствует (понятно, что моченые засекут сразу же, но смысл в том, чтобы не причинить немедленных неудобств обывателям).

Какие будут последствия? Что пришло в голову: полярное сжатие должно "разгладиться". Как это будет происходить? Предполагаю, будут землетрясения, а какого масштаба, уровня "лол, чот земля дрожит" или "мы все умрем"? Вулканы активизируются? Возникнут ли ураганные ветры, или мы карусель достаточно медленно остановили? Как быстро будет нагреваться дневное полушарие и остывать ночное (день теперь равен году) и до каких температур?
Аноним 03/09/20 Чтв 15:57:27 598712409
>>598710
Полный пиздос биосфере, полный пиздос обоим полушариям, а на терминаторе пиздецовые ураганные ветра.
Это убивает краба.
Аноним 03/09/20 Чтв 16:01:23 598715410
>>598712
>на терминаторе пиздецовые ураганные ветра
Это из-за нагрева? Типа, расширяющийся воздух с горячей стороны несется на место скукожившейся остывшей атмосферы на ночной?
Аноним 03/09/20 Чтв 17:29:06 598734411
Orbital-ATK-Peg[...].jpg 93Кб, 647x485
647x485
PegasusOrbital0[...].jpg 38Кб, 879x485
879x485
>>598641
Ну это уже обычная ракета, у такой задачи как раз есть. Но с коммерческими нагрузками у дорогущего водородного сверхтяжа тоже не особо.
Ты только что Буран. Но если ты имеешь в виду ступень с воздушными движками, а не ракетными - то ты только что воздушный старт, который применяется. Правда профитов у него мало.

ВРД не применяют как полноценный двигатель первой ступени потому, что они эффективны лишь в небольшом параметре высот и скостей, быстро летать не могут, тягу нормальную на протяжении всей траектории хуй дадут.
Аноним 03/09/20 Чтв 17:36:36 598736412
>>598734
почему они кстати пускают эту хуйню с такого редкого гражданского самолета, а не с какого-нибудь б-52?
Аноним 03/09/20 Чтв 17:40:07 598737413
image.png 58Кб, 254x199
254x199
image.png 256Кб, 623x459
623x459
>>598736
А они и запускали с б-52.
Мне кажется, что тристар дешевле и проще в обслуживании, чем военный бомбардировщик.
Ну и плюс триджеты быстро стали ненужным говном после смены разрешений на полеты над атлантикой, когда двуджеты смогли летать.
Аноним 03/09/20 Чтв 17:49:25 598738414
>>598736
Похуй что переделывать. А Б-52 бы не уложился в массогабариты (?), хуй знает на самом деле. Внешняя подвеска все равно нужна была бы. Ну и военный самолёт, а это коммерческая контора, с пегасуса вообще прибыли ждали.
Мясищевцы как-то предлагали пилить хуйню, в которой ракета засовывается прямо в трюм немодифицированного Ан-124 и выкакивается по рельсам в полёте через задний люк, крича "за ДВД!". Интересно было бы посмотреть на такую наркоманию, конечно.
Аноним 03/09/20 Чтв 17:51:58 598739415
170531-stratola[...].jpg 125Кб, 1200x675
1200x675
>>598738
Ну и да, вот этот адовый пиздец, который так ничего и не запустил. Интересно, какой ебли стоило не дать двум фюзеляжам разлететься в разные стороны.
Аноним 03/09/20 Чтв 17:54:33 598742416
Стикер 63Кб, 250x250
250x250
>>598739
Мне в огурцаче подобную хтоническую конструкцию приходилось обмазывать стяжками что твой Флайер Райтов.
Аноним 03/09/20 Чтв 18:00:16 598745417
image.png 906Кб, 864x527
864x527
image.png 12207Кб, 3030x2606
3030x2606
>>598738
>А Б-52 бы не уложился в массогабариты (?), хуй знает на самом деле. Внешняя подвеска все равно нужна была бы.
но с него еще
но мог же. с него и сейчас пускают всякие тесты
>>598737
но как может быть проще самолет, который нигде не летает? я понимаю что он им достался скорее всего бесплатно, но обслуга уникального на сегодняшний день самолета явно дороже чем обычного военного
Аноним 03/09/20 Чтв 18:02:01 598746418
image.png 12207Кб, 3030x2606
3030x2606
>>598738
>А Б-52 бы не уложился в массогабариты (?), хуй знает на самом деле. Внешняя подвеска все равно нужна была бы.
но с него еще
но мог же. с него и сейчас пускают всякие тесты
>>598737
но как может быть проще самолет, который нигде не летает? я понимаю что он им достался скорее всего бесплатно, но обслуга уникального на сегодняшний день самолета явно дороже чем обычного военного
Аноним 03/09/20 Чтв 18:05:58 598747419
image.png 891Кб, 1200x646
1200x646
>>598745
>но как может быть проще самолет, который нигде не летает? я понимаю что он им достался скорее всего бесплатно, но обслуга уникального на сегодняшний день самолета явно дороже чем обычного военного
Не, тут похую. У Траволты, частного лица, боенх 707, который уже лет 50 не актуален. Части есть или заказываются/изготавливаются, сервисное обслуживание такое же как и у любых рейсовых самолетов по сервисной книжке квалифицированным персоналом.
Военный самолет выйдет дороже в эксплуатации и обслуживании

>>598746
Ты заебал, с первого раза понятно было.
Аноним 03/09/20 Чтв 18:06:54 598749420
>>598745
>>598746
Я думаю что дедуля 52 всё-таки обосрался бы тащить 23 с хером тонны под одним крылом, и надо было бы резать брюхо, как и сделали. Ну и жрёт он небось как не в себя.
Аноним 04/09/20 Птн 08:30:55 598892421
Достаточно ли в кометах воды и аммиака чтоб накидать на Марс океан и азотную атмосферу?
Аноним 04/09/20 Птн 14:48:43 598931422
>>598892
Если ты про "терраформирование", то там говорится именно про обстрел планеты подобными телами. Получается, что одной не хватит, а пиздануть здоровой глыбой опасно.
Аноним 04/09/20 Птн 14:50:23 598932423
>>598931
А хуле случится? Не разъебем же планету
Аноним 04/09/20 Птн 14:55:44 598933424
>>598932
Ну сначала слишком дохуя просто в космос улетит и еще дохуя на орбите планеты зависнет, уже подлететь сложнее. Потому и по чуть чуть водички подливать самый норм вариант. Ну и кометы как правило объекты не маленькие и на больших скоростях, будет просто 2 планеты, а не одна няшная. Все надо садится и считать и зависит и от планеты выбранной и от того что под рукой для обстрела есть.
Аноним 04/09/20 Птн 15:00:04 598934425
>>598931
Мне казалось, что просто спамить по северному полушарию этими кометами не крупных размеров, чтоб кора не треснула и кьюриосити с персевиренсом не опрокинуло, так они будут по днищу будущего океана бить и выделившаяся энергия согревать набирающуюся атмосферу и океан.
Аноним 04/09/20 Птн 15:00:26 598935426
>>598892
Солнечный ветер и радиация снесёт атмосферу за сто лет, так как нет магнитного поля.
Аноним 04/09/20 Птн 15:01:36 598936427
Аноним 04/09/20 Птн 15:14:05 598938428
h0xkkncd4BY.jpg 21Кб, 500x201
500x201
Вопрос глупый, но занимательный.
Вот услышал такую теорию где то, что мол большой взрыв породил несколько вселенных, не только нашу что видим.
И что есть мол параллельно развивающиеся вселенные и что типа вот эти вот 30% темной материи это оно и есть, но только почему то не видно. Что думаешь анон? Словно в сосиски на тарелке, палок сосисок много и все на одном подносе и МЫ это только одна из сосисок.
Аноним 04/09/20 Птн 15:16:08 598939429
Аноним 04/09/20 Птн 15:17:21 598940430
>>598939
Значит чья то догадка. Но все же. Такое возможно? Т.е известное о темной материи позволяет такому быть?
Аноним 04/09/20 Птн 15:18:57 598941431
Какой мощности должен быть термоядерный хлопок, чтобы сдуло ощутимую часть атмосферы. Скажем, 5%. Или это нереально?
Аноним 04/09/20 Птн 15:23:26 598942432
>>598940
Ничего не известно.
Аноним 04/09/20 Птн 15:24:19 598943433
>>598941
Выше тератонны, повредит кору и огромная часть поверхности станет лавой.
Аноним 04/09/20 Птн 15:27:14 598944434
>>598943
На обратной стороне Земли тоже всем пизда, или как-то можно выжить?
Аноним 04/09/20 Птн 15:27:18 598945435
>>598942
А ее гравитационное взаимодействие? Но может ли она представлять собой вуаль покрывая то что эту гравитацию и испускает?
Аноним 04/09/20 Птн 15:28:06 598946436
>>598944
В бункерах.
Поверхность станет практически безжизненной.
Аноним 04/09/20 Птн 15:28:58 598947437
>>598945
>>598942
>Ничего не известно.
Это не имеет предсказательной силы и не проверяемо, поэтому это ненаучный вопрос.
Аноним 04/09/20 Птн 15:29:27 598948438
>>598946
А какие-нибудь сейсмические волны бункеры не сжуют? Или все же расстояние слишком большое даже для тератонны?
Аноним 04/09/20 Птн 15:30:10 598949439
>>598947
Хорошо... А что вообще известно про темную материю кроме как то что тона есть и это так назвали?
Аноним 04/09/20 Птн 15:31:50 598950440
>>598949
То, что она собирается в кучки размером с галактику.
Аноним 04/09/20 Птн 15:32:11 598951441
>>598949
То, что она гравитирует и объясняет вращение галактик.
Аноним 04/09/20 Птн 15:33:03 598952442
>>598948
Зависит от позиции и конструкции бункера. Стабилизированный в горном мегалите бункер вроде НОРАДа выживет.
Аноним 04/09/20 Птн 15:36:17 598954443
>>598950
>>598951
И в тоже время проходит(пересекается) сквозь нашу галактику, землю и нас людей самих. Я прав? Или это про темную энергию?
Аноним 04/09/20 Птн 15:42:36 598955444
>>598954
Материя. Да, проходит.
Потому, что не взаимодействует Э/М.
Аноним 04/09/20 Птн 15:44:36 598956445
>>598955
>Э/М
Электропоезд Мытищинский?...
Аноним 04/09/20 Птн 15:48:26 598957446
Аноним 04/09/20 Птн 15:48:32 598958447
Аноним 04/09/20 Птн 15:52:36 598959448
Анон, если бы тебе сейчас предложили бы понаблюдать за каким то одним неизвестным сейчас космическим явлением и получить на него ответ, то что бы ты выбрал?
Например: что за горизонтом событий у черных дыр
Аноним 04/09/20 Птн 15:56:12 598960449
>>598959
Я бы посмотрел, что происходит за горизонтом событий пизды твоей мамаши.
Аноним 04/09/20 Птн 15:59:18 598961450
>>598960
Там все тоже самое, что и у твоей.
Аноним 04/09/20 Птн 16:00:43 598962451
>>598961
Но она же мертва.
Аноним 04/09/20 Птн 16:02:07 598963452
>>598962
Ну ты же откуда то вылез
Аноним 04/09/20 Птн 17:21:57 598968453
>>598959
Если это явление неизвестно, то откуда мне знать о нем?
Ты имел в виду "неисследованном"?
Опять же, каким образом я смогу это наблюдать?
Я бы в первую очередь хотел бы видео записать, например, или снять показания спектрографов и прочих приборов, вот это наблюдения.
Воочию всё, что я могу увидеть из астрономических явлений даже будучи бессмертным летакой - это яркий свет.
Я кроме фотонов ничего не вижу, я крайне паршивый способ наблюдения.
Аноним 04/09/20 Птн 20:35:36 598981454
Итак, аноны. У меня есть одна шизофреническая идея новая конструкция прыжкового ранца, ололо, и у меня такой вопрос: чем окислять керосин/метан? Желательно не криогеника, желательно малотоксичное то бишь, не гиперголик, ибо это само по себе значит высокую реактивность.
Аноним 04/09/20 Птн 20:50:50 598982455
>>598981
Перекись ебашь, как дедушка Кармак завещал.
Аноним 04/09/20 Птн 23:03:10 598996456
>>596078 (OP)
Но видос же хуета. Это круглое тело, значит достаточно массивное, больше 1000км размером. А значит вести себя в таких условиях будет как жидкость а не как твердое тело. Тоесть сначала будет происходить деформация - растягивание сферы в каплю. Изза такой деформации будет происходить нехуевый нагрев до тысяч градусов с полным расплавлением тела, и только потом распидорашивание на мелкие куски.
Аноним 04/09/20 Птн 23:37:11 599002457
>>598981
прыжковый ранец на ракетных двигателях уже говно затея
Аноним 04/09/20 Птн 23:39:37 599003458
Чем ограничен радиус детектирования слияний ЧД в гравитационном детекторе? Можно ли задетектить слияние на существующих детекторах произошедшее прям у кромки реликтового излучения? Частота ведь та же или нет?
Аноним 05/09/20 Суб 02:13:01 599005459
>>599003
Дело не в частоте.
Аноним 05/09/20 Суб 03:03:03 599006460
Допустим, космонавт обосрется в космос. Возможно ли такое, что покак разнесет по разным телам и из покака разовьется жизнь в пригодных условиях?
Аноним 05/09/20 Суб 03:06:24 599007461
>>599006
О, давно не было. Где пропадал?
Аноним 05/09/20 Суб 03:54:11 599008462
>>597051
Слышал где-то что консистенция майонеза такая какая есть из-за квантовых эффектов, но не знаю насколько это правда.
Аноним 05/09/20 Суб 03:56:47 599009463
>>597051
Цвета определяются квантовыми эффектами вроде. Все атомы излучают на определенных длинах и именно потому, что излучают дискретные кванты определенной энергии, а не любую хуйню.
Аноним 05/09/20 Суб 08:27:42 599017464
>>599003
Нет, не такая же. Любая волна, даже гравитационная, пока ползёт по расширяющейся вселенной, тоже расширяется вместе с ней и теряет частоту и энергию.
Аноним 05/09/20 Суб 09:42:10 599020465
Есть какая-то инфа по тахионам? Есть ли какие-то явления объясняющиеся их существованием, или они чисто конструкт выведенный из математики?
Аноним 05/09/20 Суб 20:48:54 599097466
Есть какое нибудь приложение на андроид чтоб за звездным небом наблюдать как нибудь? Созвездие там найти нужное или узнать что за спутник?
Аноним 05/09/20 Суб 20:50:46 599098467
>>599097
Да, конечно есть.
Аноним 05/09/20 Суб 20:51:47 599099468
>>599098
Отлично. а какие?
Аноним 05/09/20 Суб 21:43:22 599105469
Аноним 06/09/20 Вск 08:01:00 599134470
>>599105
Я благодаря ему разглядел как-то случайно паровозик старлинков. Но звезды и созвездия там так, для ориентировки - оно заточено именно под спутники больше.
Аноним 06/09/20 Вск 08:02:26 599136471
>>598981
>Желательно не криогеника,
Селитру расплавь, лол.
Аноним 06/09/20 Вск 09:43:12 599140472
Имеет ли смысл подогревать топливо перед отправкой в камеру сгорания?
Аноним 06/09/20 Вск 10:23:48 599145473
>>599140
Оно и так подогревается. Идёт вокруг сопла. Регенеративное охлаждение ептить
Аноним 06/09/20 Вск 11:11:30 599152474
>>598981
>желательно малотоксичное то бишь, не гиперголик, ибо это само по себе значит высокую реактивность.
Малотоксичные гиперголики существуют. Вот этот >>599136 недалек от них. >>593614 →
Аноним 06/09/20 Вск 11:33:41 599154475
>>599020
>Есть ли какие-то явления объясняющиеся их существованием
Нет. Тахионы были придуманы физиками-теоретиками, которые баловались на тему "как могла бы выглядеть частица, двигающаяся быстрее света", невзирая на сломанную причинность и прочее подобное, выходящее за рамки мысленного эксперимента. Никакого соответствия реальному миру нет (хотя схожий матаппарат используется в теории поля)
>чисто конструкт выведенный из математики
Это.
Аноним 06/09/20 Вск 16:15:15 599198476
image.png 560Кб, 453x604
453x604
>>599154
>в теории поля
На конечной станции кондуктор осматривает вагоны и в одном видит на лавочке заснувшего студента, а рядом лежит книжка Ландау "Теория поля".
Кондуктор будит студента: "Ну вставай, агроном, приехали!"
Аноним 06/09/20 Вск 18:42:36 599219477
>>599152
Если гиперголик нетоксичный, проверьте его импульс условия работы.
Аноним 06/09/20 Вск 20:26:22 599229478
Бля, только сейчас узнал, что газовые гиганты РЕАЛЬНО ГАЗОВЫЕ и у них нету поверхности, мне это аж голову вскружило, до сих пор в башке бардак. Напишите, пожалуйста, годные каналы/сайты про космос на русике.
И пару вопросов!
1) Почему тот же Юпитер держит свою форму и газ с неё нахуй не улетает? Гравитация? А что именно притягивает и держит на одном месте? Как всё это облако вообще по космосу передвигается и не распадается?
2) Почему чорную дыру нельзя увидеть?
Заранее спасибо!
Аноним 06/09/20 Вск 20:34:46 599230479
>>599229
>Почему тот же Юпитер держит свою форму и газ с неё нахуй не улетает? Гравитация?
А хули с Земли воздух не улетает? Да, гравитация. Гравитирует любое вещество, в том числе и газ. Просто слабее, потому что он не такой плотный, то есть у него меньше масса. Но ведь его дохуя - так что какая разница? Солнце вон вообще из газа (точнее плазмы) состоит и ничего, все здоровы, Солнце не разваливается. Но если тебе будет от этого легче, у Юпитера есть твердое ядро.

2) Почему чорную дыру нельзя увидеть?
Потому что даже свет с нее улететь и вонзиться тебе в глаз не может. Но можно увидеть аккреционный диск вокруг нее, например. Или гравитационное линзирование.
Аноним 06/09/20 Вск 20:38:50 599231480
>>599229
>газовые гиганты РЕАЛЬНО ГАЗОВЫЕ и у них нету поверхности
У них нету поверхности в том смысле, что там идет плавный переход от газа к жидкости к твердому веществу. Самое обидное, что внутри Юпитера лежит в огромных количествах охуеннейшее ракетное топливо - металлический водород, но его оттуда никак не вытащить.
Аноним 06/09/20 Вск 20:42:16 599232481
>>599231
Даже если б был способ, дешевле сжать водород заново, чем ковырять тысячи километров Юпитера.
Аноним 06/09/20 Вск 21:00:29 599234482
>>599230
>А хули с Земли воздух не улетает? Да, гравитация. Гравитирует любое вещество, в том числе и газ. Просто слабее, потому что он не такой плотный, то есть у него меньше масса. Но ведь его дохуя - так что какая разница? Солнце вон вообще из газа (точнее плазмы) состоит и ничего, все здоровы, Солнце не разваливается. Но если тебе будет от этого легче, у Юпитера есть твердое ядро.
Просто удивляет, что по космосу летает охуенно огромный сгусток газа и ничего с ним не происходит. Почему именно такие размеры, почему не больше, почему не меньше? Потому что сила гравитации определённая и не позволяет всему этому улетать или, наоборот, увеличиваться?
А что вообще излучает эту гравитацию?

>Потому что даже свет с нее улететь и вонзиться тебе в глаз не может. Но можно увидеть аккреционный диск вокруг нее, например. Или гравитационное линзирование.
Огромный чёрный сгусток в космосе, который что-то засасывает, не может намекать на дыру? Или вообще не так выглядит? Или такое часто случается, мол, просто пространство без планет?

>>599231
>У них нету поверхности в том смысле, что там идет плавный переход от газа к жидкости к твердому веществу.
К жидкости? Как это вообще выглядит? Если у нас жидкость просто расположена на поверхности, то там просто в воздухе? Просто представить не могу, что на ядре Юпитера расположено дохуища, в несколько раз больше, чем ядро, жидкости. Или не больше?
Аноним 06/09/20 Вск 21:11:04 599235483
>>599234
>Почему именно такие размеры, почему не больше, почему не меньше?
Больше может быть, причем намного (те же звезды), меньше - вряд ли (если мы не о твердом веществе). И Юпитер без ядра, скорее всего, не смог бы образоваться. То есть, при нынешнем размере твердое ядро ему и не нужно, он бы и так был стабильным. А вот накопить такую массу начиная с газа не получится. Почитай вот это, хотя это больше про звезды, но ближе к тому, что тебя интересует, как я понимаю:
https://ru.wikipedia.org/wiki/Гравитационная_неустойчивость

>Огромный чёрный сгусток в космосе, который что-то засасывает, не может намекать на дыру?
Он нихуя не огромный, ЧД с массой Солнца имела бы радиус порядка 3 км, мы такого размера даже не все астероиды в Солнечной системе знаем (причем мы их засекаем по отраженному свету, а ЧД не отражает). Все известные ЧД от нас дохуя далеко, с такого расстояния они ничем от точки не отличаются. Если б ты был на расстоянии, с которого ЧД выглядит огромной, тебя бы уже убило. Да и вообще, космос довольно черный в среднем.
Аноним 06/09/20 Вск 21:51:48 599241484
Аноны, а что сейчас с воздушным стартом? С одной стороны размеры спутников падают - часто запускают всякую мелочь. С другой - наши вояки (а еще китайский и США) двигают тазом, всячески изобретая разный гиперзвук и противоспутниковые ёбы, которые как раз по идее и должен запускаться с больших высот.
Нет мыслей объединить или пока все одно дороже обычного пуска выходит?
Аноним 06/09/20 Вск 22:04:30 599243485
>>599241
а смысл? военным нужна мобильность, в обычных космических запусках хз зачем это нужно
Аноним 06/09/20 Вск 22:24:16 599244486
Сколько таких же солнечных систем в млечном пути, как наша?
И что в центре галактик? Такое яркое и большое.
Аноним 06/09/20 Вск 22:28:30 599245487
>>599244
>Сколько таких же солнечных систем в млечном пути, как наша?
Насколько таких же? Вообще звезд несколько сотен миллиардов, но есть же и дохуя непохожих звезд среди них. А у похожих могут быть непохожие системы.
>И что в центре галактик?
Обычно сверхмассивные черные дыры.
>Такое яркое и большое.
Много звезд. Иногда (особенно в относительно молодых галактиках, которые далеко от нас) СМЧД много кушают и светят, что дурные, но обычно это просто много-много звезд, сидящих (относительно) близко друг от друга.
Аноним 06/09/20 Вск 22:40:05 599247488
>>599245
>Насколько таких же? Вообще звезд несколько сотен миллиардов, но есть же и дохуя непохожих звезд среди них. А у похожих могут быть непохожие системы.
И вокруг этих сотен миллиардов звёзд вращаются сотни миллиадов планет? Или они одиночки?
Как вообще люди узнают, как образуются, из чего состоят и каким законам подчиняются планеты/галактики далеко-далеко-далеко от нашей? Как люди узнают о внешности таких далёких планет, дорисовывают очертания или телескопы настолько мощные?
Аноним 06/09/20 Вск 22:49:25 599248489
>>599247
>И вокруг этих сотен миллиардов звёзд вращаются сотни миллиадов планет? Или они одиночки?
Ну, в среднем открытых планет на звезду довольно мало. Но вполне вероятно, что это связано в первую очередь с недостатками методов обнаружения. Короче, пока рано говорить, но скорее всего 1-2 планета на звезду уж точно будет (может и значительно больше).
>Как вообще люди узнают, как образуются, из чего состоят и каким законам подчиняются планеты/галактики далеко-далеко-далеко от нашей?
Основной физический принцип: изотропность пространства, то есть что физические законы везде одинаковы. Он принципиально недоказуем, но отлично соответствует всем наблюдаемым явлениям (то есть, опровержений не наблюдается) и общенаучно-философскому принципу бритвы Оккама.

Состав планет определяется спектрометрией, отчасти следует из плотности (если плотность 1 г/см^3, то явно там не железо), хотя для экзопланет это редко можно применить. Из общих теорий гравитации и планетообразования.
>Как люди узнают о внешности таких далёких планет, дорисовывают очертания
Методом тыка, обычно такой хуйней собственно моченые не занимаются. Посмотри на Юпитер, есть основания ожидать, что планеты со сходными параметрами будут выглядеть похоже.
>или телескопы настолько мощные?
Нет. Даже диски удаленных звезд видны в единичных случаях (с крошечным разрешением). О наблюдении диска экзопланет пока и речи нет. Они и открываются в подавляющем большинстве случаев (или во всех?) не прямым наблюдениям, а по "косвенным уликам".
Аноним 06/09/20 Вск 23:04:23 599249490
>>599248
>Ну, в среднем открытых планет на звезду довольно мало. Но вполне вероятно, что это связано в первую очередь с недостатками методов обнаружения. Короче, пока рано говорить, но скорее всего 1-2 планета на звезду уж точно будет (может и значительно больше).
Планеты тяжело находят из-за их блеклости? Хоспади, если только на 1 галактику несколько сотен миллиадов звёзд, то насколько же огромна вселенная в целом, пиздец просто...
>Нет. Даже диски удаленных звезд видны в единичных случаях (с крошечным разрешением). О наблюдении диска экзопланет пока и речи нет. Они и открываются в подавляющем большинстве случаев (или во всех?) не прямым наблюдениям, а по "косвенным уликам".
То есть человечество знает ничтожно малое количество и состав галактик? Часто видел, как появлялась инфа про чёрные дыры с планетами через овер9000 световых лет, это тоже предположения? Тот снимок чёрной дыры за 50 млн световых лет, это сильно дохуя или дыра сильно аномальна?
Спасибо за ответы!
Аноним 06/09/20 Вск 23:10:44 599250491
>>599249
>Планеты тяжело находят из-за их блеклости?
Ну, формально да, а также за счет близости к звезде: банально не хватает разрешающей способности различить тусклую точку планеты возле звезды, более яркой на порядки. Но на самом деле прямым наблюдением считай что нихуя не открывают, там все сложнее: https://ru.wikipedia.org/wiki/Экзопланета#Методы_поиска_экзопланет
>То есть человечество знает ничтожно малое количество и состав галактик?
Как раз состав галактик узнать проще: они светятся, так что можно применить спектрометрию.
>Тот снимок чёрной дыры за 50 млн световых лет, это сильно дохуя
Да, там какая-то магия по сведению снимков с телескопов на разных "концах" Земли для получения одного виртуального телескопа в пол-Земли размером. Не спрашивай, я сам тут толком нихуя не понимаю.
Аноним 06/09/20 Вск 23:30:26 599251492
>>599250
И последнее. Что будет, если Юпитер столкнётся с обычной планетой, вроде Земли, и другим газовым гигантом, вроде Сатурна?
Аноним 07/09/20 Пнд 00:08:12 599255493
>>596078 (OP)
Подскажите документалок с крутыми визуализациями наподобие оп вебмки, но не старые которые еще до 2010 снимали. BBC Планеты все пересмотрел.
Аноним 07/09/20 Пнд 00:54:12 599259494
Waterphasediagr[...].gif 123Кб, 1844x1246
1844x1246
>>599234
>К жидкости? Как это вообще выглядит? Если у нас жидкость просто расположена на поверхности, то там просто в воздухе?
Фазовую диаграмму представляешь? Просто при определенном давлении и температуре различие между газом и жидкостью пропадает.
https://ru.wikipedia.org/wiki/Сверхкритическая_жидкость
На поверхности Венеры атмосфера находится в том же состоянии, там литералли сверхкритический океан-неокеан из углекислого газа.

>А что вообще излучает эту гравитацию?
Любое тело с массой, даже ты. "Излучает" - неправильное слово, не надо вскрывать эту тему...

>>599231
>Самое обидное, что внутри Юпитера лежит в огромных количествах охуеннейшее ракетное топливо - металлический водород, но его оттуда никак не вытащить.
Блять, у него свойства-то изучены уровня теоретическими писями на воде виляно, для крошечной искусственно созданной в лаборатории песчинки между двумя алмазными наковальнями, о которой до сих пор спорят, нужная ли это фаза. Не надо путать подобные исследования с фактами, и уж тем более не надо делать из этой субстанции ракетное топливо, с таким же успехом можно хотеть звезду отбуксировать в виде источника энергии для движка, а чё - можно же.
Аноним 07/09/20 Пнд 01:05:10 599260495
>>599259
>не надо делать из этой субстанции ракетное топливо
Так он же без юпитерианского давления просто станет газообразным, не? Тоже дело. Другой вопрос как его оттуда высосать и зачем нам СТОЛЬКО водорода именно оттуда, когда это, мягко говоря, далеко не самый редкий элемент в системе.
мимо-анон
Аноним 07/09/20 Пнд 01:07:04 599261496
>>599260
Да просто есть предположение (пока толком ничем не обоснованное, просто теорию крутят), что оно может оказаться метастабильным и не раскукожиться взрывным способом. Недавно статья была.
Аноним 07/09/20 Пнд 01:40:31 599263497
>>599261
>оно может оказаться метастабильным
Но Холмс, если оно метастабильно, то почему же мы его нигде не наблюдаем?
Аноним 07/09/20 Пнд 02:00:23 599264498
Аноним 07/09/20 Пнд 03:03:55 599265499
>>599251
>И последнее. Что будет, если Юпитер столкнётся с обычной планетой, вроде Земли, и другим газовым гигантом, вроде Сатурна?
1. Земля поглотит Юпитер, сделав его своей атмосферой.
2. Сатурн и Юпитер столкнувшись разлетятся газовыми осколками. На земле возможно будет наблюдать метеоритный дождь из газовых астероидов.
Аноним 07/09/20 Пнд 03:45:06 599268500
>>599230
>Солнце вон вообще из газа (точнее плазмы) состоит и ничего, все здоровы, Солнце не разваливается.
Эт наше. А соберешь звезду массой выше 150 солнечных масс, и каюк.
Аноним 07/09/20 Пнд 04:47:54 599272501
>>599263
Ну, может оно при нагреве раскукоживается, а все природные процессы добычи его из ядра гигантов (типа столкновения с другим гигантом или разрывом его аккреционными силами у ЧД) оным нагревом обязательно сопровождаются.
Аноним 07/09/20 Пнд 04:50:51 599273502
>>599247
>И вокруг этих сотен миллиардов звёзд вращаются сотни миллиадов планет?
Скорей всего.
>Как вообще люди узнают, как образуются, из чего состоят и каким законам подчиняются планеты/галактики далеко-далеко-далеко от нашей?
Science, bitch!
>Как люди узнают о внешности таких далёких планет, дорисовывают очертания или телескопы настолько мощные?
Предположения.
Аноним 07/09/20 Пнд 05:49:25 599276503
>>599272
С учетом температуры и давления в этих самых ядрах гигантов там должна получаться прессованная плазма. Насколько она обладает металлическими свойствами - интересный вопрос для олимпиады по химии, но как британские ученые собираются удерживать её при комнатной температуре - загадка.
Аноним 07/09/20 Пнд 09:22:50 599296504
>>599263
Потому что его никто не достал из Юпитера, очевидно же. А больше нигде в природе таких давлений нет.
Аноним 07/09/20 Пнд 12:58:34 599373505
Если наша галактика столкнётся с другой, то мы ВСЁ?
Миллиарды лет в ращоты брать не будем
Аноним 07/09/20 Пнд 13:00:24 599375506
image.png 3108Кб, 1280x1600
1280x1600
>>599373
Не, похую, максимум систему выкинет из галактики, но нам будет ВСЁ из-за Солнца самого.
Аноним 07/09/20 Пнд 13:29:02 599381507
>>599373
Нам будет похуй абсолютно. Вероятность прямого столкновения исчезающе мала, а для того чтобы нарушить планетарную систему, мимозвезде нужно практически внутри пролететь. Что тоже исчезающе маловероятно. Ну и столкновение галактик - процесс пиздец какой медленный.
Аноним 07/09/20 Пнд 13:31:56 599383508
>>599381
>>599375
А если квазар загорится в получившейся йоба-дыре, он не стелиризует галактику?
Аноним 07/09/20 Пнд 13:39:44 599384509
>>599383
Зависит от того как далеко пролететь от него, и какой он светимости. Прям всю галактику - нет.
Аноним 07/09/20 Пнд 14:07:01 599387510
>>599381
Ну, ващет там еще может астероидная бонбардировка начаться из-за относительно близкого прохождения звезды в облаке Оорта, которая нарушит орбиты его тел. Еще, считается что Солнце удачно расположено в области Галактики, где мало молодых взрывающихся звезд, что благоприятно для стабильной жизни, а это может весьма нарушиться при столкновении. Но да, это все из разряда "вероятность рака увеличилась в два раза", а не немедленных катастроф.
Аноним 07/09/20 Пнд 14:08:31 599388511
>>599387
>нарушиться
русский выучи сперва
Аноним 07/09/20 Пнд 14:09:47 599389512
>>599388
может [что сделать?] - нарушиться
Теперь ты.
Аноним 07/09/20 Пнд 14:10:55 599391513
>>599389
нет, меня постоянно хуесосили и я запомнил что мягкий знак может ставится только после шипящих типа ш и щ
Аноним 07/09/20 Пнд 14:16:59 599392514
Аноним 07/09/20 Пнд 14:26:54 599394515
Аноним 07/09/20 Пнд 15:21:43 599396516
Можно ли как0то разъебать Юпитер? Условно влетит в него астероид размером с Землю, то он потеряет свою форму? Или поведёт себя как облачко, а затем обратно соберётся?
Аноним 07/09/20 Пнд 15:23:22 599397517
>>599396
>астероид размером с Землю
Эт планета уже.
>то он потеряет свою форму? Или поведёт себя как облачко, а затем обратно соберётся?
Смотря с какой скоростью. На второй влетит - просрется красиво, но целый останется. На 0.99с - на соседних звездных системах новое светило в небе появится.
Аноним 07/09/20 Пнд 15:28:58 599398518
>>599397
>на соседних звездных системах новое светило в небе появится.
Да ну, юпитер не загорится даже если в него жмякнуть землей на световой скорости. Просто ебнет от релятивистских энергий в макромасштабах, без поддержания реакции.
Аноним 07/09/20 Пнд 15:38:11 599399519
>>599398
У атомов земли по 70ГэВ каждого будет ~ 1e-8 джоулей.
На земле 1050 атомов примерно.
Итого 10е42 джоуля испускаемые за секунду.
Это перекрывает 10е26 ватт солнца на 16 порядков.
Более того, это сопоставимо со сверхновой, но не так продолжительно.
Это полностью испарит все планеты солнечной системы сдув большую часть солнца и останется разлетающимся тускнеющим молекулярным облаком плазмы.

Я не говорил, что звезда будет гореть долго.
Аноним 07/09/20 Пнд 15:41:48 599400520
>>599399
Что-то многовато. А кинетическая энергия осколков? Не все же излучится.
Аноним 07/09/20 Пнд 15:52:06 599401521
>>599400
Можешь смело два порядка отбросить, всё равно 1% прореагировавших полностью атомов это 10е40 джоулей.
Аноним 07/09/20 Пнд 16:03:59 599403522
>>599399
Я и говорю - ебнет. А энергии +- такого порядка возникают всегда, когда мы начинаем швыряться объектами макромира на световых скоростях.
Аноним 07/09/20 Пнд 16:04:01 599404523
>>599399
Т.е. если ёбнет по Юпитеру, то сдохнут все?
Аноним 07/09/20 Пнд 16:05:56 599405524
>>599404
Мы - да. Даже если не на той земле, которой ебнули по Юпитеру, а на обычной.
Аноним 07/09/20 Пнд 16:10:13 599406525
>>599404
Тебе же сказали, от скорости зависит. На минимальной будет плюх, клочья газа и новое ядро у юпитера - у него даже радиус особо не вырастет, насчет орбиты не уверен, но скорее всего тоже не особо поменяется. Про 0,99с анон выше все правильно написал.
Аноним 07/09/20 Пнд 16:12:28 599408526
>>599406
Я себя дополню тут - еще и от траектории зависит.
Обычно практически никогда ничего целиком не плюхается отвесно, а значит, что планета попадет в полость Роша и красочно огромными ошметками выпадет в атмосферу Юпитера.
Это тоже посчитать можно, тащемта.
Аноним # OP 07/09/20 Пнд 16:12:59 599409527
laplace.jpg 14Кб, 280x336
280x336
Аноним 09/09/20 Срд 12:58:31 599874528
>>599249
>какая огромная вселенная
А какая разница, если мы там все равно никогда не будем. Скорость света и расширение вселенной не даст нам когда-либо покинуть галактику.
Алсо, чтобы охуеть от размеров вселенной и графония рекомендую SpaceEngine, крутая штука.
Настройки X
Ответить в тред X
15000
Макс объем: 40Mб, макс кол-во файлов: 4
Кликни/брось файл/ctrl-v
Стикеры X
Избранное / Топ тредов